Пределы в математике для чайников: Недопустимое название — Викиучебник

Содержание

Что это такое Предел (математика). Энциклопедия

Пользователи также искали:

предел функции определение, пределы формулы, пределы с корнями, задачи на пределы с решениями, замечательные пределы, Предел, пределы, предел, функции, решениями, задачи на пределы с решениями, пределы с корнями, предел функции на бесконечности, примеры, самостоятельного, решения, замечательные пределы, замечательные, бесконечности, определение, формулы, второй, замечательный, онлайн, задачи, корнями, пределы формулы, предел функции определение, математика, Предел математика, второй замечательный предел онлайн, пределы примеры для самостоятельного решения, предел (математика), пределы.

предел (математика),

Высшая математика для чайников. Предел функции_Виосагмир И.А_2011 -88с

Высшая математика для чайников. Предел функции

2011 год

89

’89

На рисунке представлены две функции:

 

•O и

E•O .

Как видите, они очень похожи, поэтому очень важно, запомните Вы их или нет. Давайте проведем небольшой опыт. Попробуйте запомнить два графика. Как только будете уверены в том, что все выучили, прорешайте все пределы ниже, а потом проверьте себя по графикам.

№1. Посчитать пределы:

 

 

 

 

 

 

 

 

 

 

arcsin

 

arccos

 

 

 

 

 

 

 

 

 

 

 

функции

 

.

обратная

функция

к

 

 

 

 

 

 

arcsin

 

 

 

 

 

 

 

 

 

 

 

 

 

 

sin

 

обратная

функция

к

 

 

 

 

 

функции

 

.

 

 

 

 

 

 

 

 

 

 

 

 

 

arccos

 

 

 

 

 

 

 

 

 

 

 

 

 

 

cos

 

 

 

 

 

 

 

 

 

 

 

 

№1.

Посчитать предел:

 

 

 

 

 

 

 

 

 

 

 

 

 

 

 

 

 

 

 

 

 

 

 

 

 

 

 

 

 

 

 

 

 

 

lim arcsin .

 

→ 0

 

 

Давайте посмотрим на график

 

 

 

 

. Что мы видим? При

функция принимает

 

 

 

 

 

 

Например,

 

 

 

и

 

и т.д. Делаем

бесконечно много значений.

 

 

arcsin

 

 

 

 

вывод: у нашего графика есть период.

 

lim→ arcsin

0 lim→ arcsin ‚

 

 

 

 

 

 

 

w целое число, лежащее в промежутке ∞, ∞

 

 

lim arcsin ‚w,

 

 

 

 

 

 

 

 

 

 

30

 

 

 

 

 

Высшая математика для чайников. Предел функции

 

 

То же самое с

2011 год

arccos .

 

 

arctg

 

 

 

 

 

 

 

 

 

 

 

 

 

 

 

 

 

 

arcctg

 

 

 

 

 

 

 

 

 

 

 

 

 

 

 

 

 

arctg

 

 

 

 

 

 

 

 

 

 

 

 

 

 

 

 

 

 

 

 

 

tg

 

 

– обратная функция к

 

 

 

 

 

 

 

 

 

 

 

 

 

 

 

 

 

 

 

 

 

 

 

 

 

 

 

 

 

 

 

 

 

 

 

 

функции

 

.

 

 

 

 

 

 

 

 

 

 

 

 

 

 

 

 

 

 

 

 

arcctg

 

 

 

 

 

 

 

 

 

 

 

 

 

 

 

 

 

 

 

 

ctg

 

 

– обратная функция к

 

 

 

 

 

 

 

 

 

 

 

 

 

 

 

 

 

 

 

 

 

 

 

 

 

 

 

 

 

 

 

 

 

 

 

 

 

функции

 

 

.

 

 

 

 

 

 

 

 

 

 

 

 

 

 

 

 

 

 

№1. Посчитать предел:

lim

arctg

w ∙ 2

 

 

 

 

 

 

 

 

 

 

 

 

 

 

 

 

 

 

 

 

 

 

 

 

w

 

 

 

 

 

 

 

 

arctg

 

 

 

 

 

 

 

 

целое число, имеющее шаг 2. Т.е.

lim

 

 

 

 

. Можно записать

 

 

 

 

 

 

 

 

 

 

 

 

 

 

 

 

 

 

 

 

 

 

 

 

 

 

 

 

 

 

 

 

 

 

 

 

 

 

вот так:

 

 

 

 

lim arctg

2

ˆ

2 2 w

 

 

 

 

 

 

 

 

 

 

 

 

 

 

 

 

 

 

 

 

 

 

 

 

 

 

 

 

 

 

 

‚ ‚

 

 

 

 

 

 

 

 

 

 

 

 

 

 

 

 

 

 

 

 

 

 

 

 

 

 

 

 

 

 

Заметим, что ‰ это произвольное целое число, которое мы задаем сами.

 

 

На этом, мы заканчиваем наш раздел – графики элементарных функций.

От автора:

Поздравляю! Вы смогли завершить первую главу “Предел функции” первой части “Предел и непрерывность функции”. Конечно, это не все. Я рассказал Вам лишь элементарные вещи. Далее нас будут ждать первый замечательный и второй замечательный приделы и другие методы взятия пределов. Если Вы поняли все, что я здесь написал, то дальше будет только интересно! Ничего сверхсложного вас не ожидает…

31

Высшая математика для чайников. Предел функции

2011 год

Глава 2. Непрерывность функции в точке.

Содержание:

1)Непрерывность функции в точке

2)Непрерывность сложной функции

3)Классификация точек разрыва

4)Непрерывность элементарных функций

5)Первый замечательный предел

6)Второй замечательный предел

7)Кратко о Maple

1. Непрерывность функции в точке.

Функция называется

непрерывной в точке a, если

lim

Запомните это определение раз и навсегда! Если вы его не знаете, вы – ничто и никто в

математике. Давайте рассмотрим простой пример:

1

 

 

 

 

 

 

 

 

 

 

1; 0.

Задание: проверить функцию на непрерывность в точках

1.

1. Используя определение 1, получаем:

 

1

 

 

1

1 1

1

 

lim

 

1

 

Выполняется определение 1? Да!

 

 

1

1

 

 

 

 

lim

 

 

 

 

1

1.

 

 

 

 

Вывод: функция непрерывна в точке

 

 

 

 

2.

0. Используя определение 1, получаем:

 

1

 

 

1

∞ 0

 

lim

 

0

 

Выполняется определение 1? Нет!

 

lim

0

 

 

 

 

 

 

 

 

 

 

 

 

1

 

 

 

 

 

 

 

 

32

 

 

 

Высшая математика для чайников. Предел функции

2011 год

Вывод: функция не существует в точке 0.

Пусть функция определена в правой (левой) полу окрестности точки a, т.е. на некотором полуинтервале , & (соответственно

, ).

Функция

называется

непрерывной справа

(соответственно слева) в точке a, если

 

 

lim

 

>соответственно

lim

E.

 

 

 

Здесь то же самое. Пожалуйста, рассмотрите сами такие функции как ln , и другие. Хотя, думаю, что все предельно ясно.

Для того чтобы функция была непрерывна в , необходимо и достаточно, чтобы она была непрерывна в этой точке справа и слева.

 

O

,

 

,

O

 

также непрерывны

в точке

 

 

O

 

 

 

O /O

 

 

 

 

 

 

Если функции

и

 

непрерывны в точке

, то функции

 

,

 

O 0

 

 

 

 

 

 

 

 

 

(частное – при

 

 

 

 

 

 

 

 

 

 

 

условии

 

).

 

 

 

 

 

 

 

 

 

 

 

Пример №1.

Исследовать на непрерывность функцию

 

 

 

 

 

 

.

 

 

Для начала распишем область определения

 

 

 

 

, т.к. знаменатель не

 

 

 

 

 

теорему 6:

 

 

может равняться 0. Теперь просто используем D

 

∞, 0 0, ∞

 

 

 

 

lim

 

 

,

 

 

 

0

 

 

 

 

 

 

0

 

 

 

 

где

 

 

 

 

 

 

непрерывна в любой точке,

 

 

. Следовательно, по теореме 6, функция

 

кроме

 

 

.

 

 

 

 

 

 

 

 

 

 

 

 

33

Высшая математика для чайников. Предел функции

2011 год

2. Непрерывность сложной функции.

Пусть функция F определена на множестве , а G множество

значений этой функции. Пусть, далее, на

множестве

G

определена

функция H . Тогда говорят, что

на

множестве

 

определена

сложная функция, и пишут H , где

F, или H F.

Впрочем, пока что вам это не сильно понадобиться. Привожу примеры сложных функций:

1

,

log

 

 

 

1 .

b|sin |, cos

 

 

Почему они сложные? Давайте рассмотрим цепочку последовательных преобразований для первой из них:

sin •

|•|

å.

Вот и все! Теперь перейдем ко второй функции:

 

 

1

cos •

•.

И так далее. Не хочется уделять этому много времени. Надеюсь, вы и так все поняли. Ну что же, перейдем к теореме.

непрерывна в

 

 

 

 

 

 

 

 

непрерывна в точке .

 

 

 

в точке , а

P• Q

Пусть функция

 

 

 

непрерывна

 

функция

 

 

 

точке

 

 

 

. Тогда сложная функция

 

 

 

 

 

 

 

 

 

 

 

 

Давайте рассмотрим пример на доказательства. Здесь как раз и нужно рассматривать сложную функцию.

Пример №1

 

 

 

Доказать, что:

lim

 

ln , 0, 1.

 

Рассмотрим функцию •

1

 

1. Она непрерывна в точке 0 и 0 0. При этом

34

Высшая математика для чайников. Предел функции

 

 

 

 

 

 

 

2011 год

 

 

 

 

log

 

1 ,

1

 

 

 

 

 

log 1 .

Вычислим lim→

 

:

 

 

 

 

 

 

 

 

 

 

 

 

 

 

ln

 

 

 

 

 

 

 

lim log 1

lim ln 1

Этот шаг может быть непонятен, поэтому я должен напомнить вам формулу преобразования к логарифму с другим основанием:

 

 

 

 

 

 

 

 

 

 

 

log

log

 

 

 

 

 

 

 

 

 

 

 

 

 

 

 

 

 

 

 

 

 

 

 

log

 

 

 

 

 

 

 

 

 

 

 

 

 

Запомните ее и больше не возвращайтесь к этому. В данном случае

 

 

 

новое основание.

Давайте напишем формулу именно для нашего случая:

 

ln 1

 

 

 

 

 

 

 

 

 

 

 

 

 

 

 

log

 

1

 

 

log 1

 

.

 

 

 

 

 

 

 

 

Итак, продолжаем:

 

 

 

 

 

log

 

 

 

ln

 

 

 

 

 

 

 

 

 

 

 

 

 

 

 

 

ln

 

 

1

 

 

 

 

 

 

 

 

 

 

 

 

 

 

 

 

 

 

 

 

 

 

 

 

 

 

 

 

 

 

 

 

 

lim log 1

 

lim ln 1

ln →

ln 1

.

 

 

 

 

 

 

 

 

 

 

 

 

 

 

 

 

 

 

 

 

 

 

 

 

 

lim

 

 

 

 

 

 

 

 

 

Верно? ln это число, поэтому мы его и вынесли. Теперь нужно посчитать предел lim→

 

.

Представим

функцию в

виде

ln 1

 

ln “

(тоже свойство

логарифма!), где

1 .

 

 

 

 

 

 

 

 

 

 

 

 

 

 

 

 

 

 

 

 

 

 

 

 

 

 

 

 

 

 

 

∙ log log

 

 

 

 

 

 

 

 

 

 

 

Так как

 

 

 

 

(Это второй замечательный предел. Пока что мы его не прошли,

 

 

 

 

 

 

 

 

 

 

 

 

 

непрерывна в точке

 

 

 

, то

 

 

 

но,

поверьте, равенство верно), а функция

 

 

 

 

 

 

 

 

 

lim

 

 

1

 

 

 

 

 

 

ln “

 

 

 

ln ’

1.

 

 

 

 

 

 

 

 

 

 

 

 

 

 

 

 

 

 

 

 

 

 

 

 

 

 

 

 

 

 

 

 

 

 

 

 

 

 

 

 

 

 

 

 

 

 

 

 

 

 

 

 

 

 

 

 

 

 

 

 

 

 

 

 

 

 

lim ln 1

 

 

 

 

 

 

 

 

 

 

 

 

Возвращаемся к нашему примеру. И вот, что у нас получается:

35

Высшая математика для чайников. Предел функции

2011 год

lim

 

 

 

= lim

 

 

ln

 

 

= ln

 

 

 

 

 

1

 

 

 

=

ln

 

= ln

 

.

 

 

 

 

 

 

 

 

 

 

 

 

 

lim

 

 

 

 

 

 

 

 

 

 

 

 

log

 

 

ln 1 +

 

 

 

 

 

ln(1 +

 

)

 

 

 

 

 

 

 

(1 + )

 

 

 

 

 

 

 

 

 

 

 

 

 

 

 

 

 

 

 

 

 

1

 

 

 

Рассмотрим теперь функцию ( ), непрерывную в точке = 0:

 

 

 

 

 

 

 

 

 

 

 

 

 

 

 

 

 

 

 

ln

 

 

 

 

 

 

 

 

при

= 0

 

 

 

 

 

 

 

 

 

 

 

 

 

 

 

 

 

 

 

 

 

 

 

(1 +

 

)

 

при

 

≠ 0

 

 

 

 

 

 

 

 

 

 

 

 

 

 

 

 

=

 

 

 

log

 

 

 

 

 

 

 

 

 

 

 

 

 

 

 

 

 

 

 

 

 

 

Согласно теореме 8 сложная функция

 

 

 

 

 

 

 

 

 

 

 

 

 

 

при = 0

 

 

 

 

 

 

 

 

 

 

 

 

 

 

P• Q

 

 

 

 

ln

 

 

 

 

 

 

 

 

 

 

 

 

 

 

 

 

 

 

 

 

 

 

 

=

 

 

 

 

− 1

при

 

 

≠ 0

 

 

 

 

 

 

 

 

Является непрерывной в точке = 0. Поэтому

 

 

 

 

 

 

 

 

 

 

 

 

 

 

 

 

 

 

 

 

 

 

 

 

 

 

 

 

 

 

 

 

 

 

 

 

 

 

 

 

 

 

.

 

 

 

 

 

 

 

 

 

 

 

 

 

 

 

 

 

 

 

 

 

 

 

 

lim

 

 

− 1 = ln

 

 

 

 

 

 

 

 

 

 

 

 

 

 

 

Сложно? Может быть, но вы должны в этом разобраться, потому что это очень важно для понимания этой темы. Тем более, здесь требуется внимательность, ну и “немножко подумать”.

36

Высшая математика для чайников. Предел функции

2011 год

3. Классификация точек разрыва.

Для начала, давайте поймем, что вообще означает “точка разрыва”. Все предельно просто!

Точка называется точкой разрыва функции , если в этой точке не является непрерывной.

Прежде чем начинать рассматривать классификацию точек разрыва, вы должны всегда проверять условие: должна быть определена в некоторой окрестности точки , за исключением, быть может, самой точки .

Если условие выполняется, то можно рассматривать классификацию точек разрыва.

Точка – устранимая точка

разрыва, если

lim #

Пример №1.

 

 

 

 

 

sin

 

 

 

 

 

 

 

 

 

 

 

 

 

 

 

 

 

 

 

 

 

 

∞; 0 0; ∞

 

 

 

0

необычная точка. В ней

 

 

D

 

 

 

 

 

 

 

 

 

 

 

 

 

Прежде всего, напишем область определения:

 

 

 

 

 

. Отсюда сразу видно,

что

 

 

 

функция

не

определена, но

 

 

определена в ее

окрестности.

→ sin

 

 

 

sin

 

 

 

 

 

 

1

0

.

 

 

 

 

 

lim

 

 

 

 

 

 

Отсюда следует, что 0 устранимая точка разрыва.

 

 

 

 

 

37

Высшая математика для чайников. Предел функции

2011 год

Точка – точка разрыва первого рода, если

lim

#

lim #

 

 

 

 

Пример №1.

 

sgn

 

 

Функция sgn уже должна быть ранее вам известна, но я вам ее напомню.

sgn

1,

0

” 0,

 

0,

 

1,

, 0

sgn

1,

 

 

 

 

 

lim

 

 

 

 

 

sgn

1,

 

 

f(x) = sgn(x)

lim

 

 

 

 

 

0

0.

 

lim→ sgn lim→ sgn sgn

 

 

0

 

Отсюда следует, что

точка

точка разрыва

первого рода.

 

 

 

Точка – точка разрыва второго рода, если хотя бы один из односторонних пределов не существует или равен бесконечности.

Пример №1.

 

tg

 

 

‚w˜ , w 0 ™.

Прежде всего, напишем область определения D

–\ 2

 

tg

 

 

 

 

 

lim

 

 

38

 

 

Высшая математика для чайников. Предел функции

2011 год

 

tg

 

 

 

 

 

lim

 

Т. к. хотя бы один из пределов равен бесконечности, то

‚w точка разрыва второго рода.

Пример №2.

 

ln

 

 

 

0; ∞ .

Прежде всего, напишем область определения D

 

 

 

0

 

 

lim ln

 

 

 

 

 

 

 

lim ln

 

 

Т.к. хотя бы один из пределов не существует, то

 

0 точка разрыва второго рода.

Итак, мы теперь знаем классификацию точек разрыва. Мы рассмотрели примеры к каждому случаю. Они достаточно легкие, поэтому давайте еще попрактикуемся. Во всех следующих номерах определить точки разрыва.

P.S. Для начала попробуйте сделать это сами, ну а потом проверьте себя. Удачи ☺!

№1.

т. 1 функция

№2.

Прежде всего, напишем

 

 

,

( 1

 

2ln ,

1

 

lim

lim

 

ln

0,

 

 

1.

lim

lim

 

 

 

lim

lim

 

 

имеет разрыв первого рода.

 

 

D

 

∞, 0 0, ∞ .

 

Цитаты про предел. Пределы в математике для чайников: объяснение, теория, примеры решений

Знай, у каждого разное «больно»,
Знай, у каждого разное «страшно».
Не суди со своей колокольни
Неизвестносколькоэтажной.

Не очерчивай взглядом границы,
Не придумывай мозгом пределы.
Что тебе в страшном сне не приснится,
Для кого-то – обычное дело.

Знай, у каждого разное «надо»,
Знай, у каждого разное «сложно».
Впрочем, и представление ада
Обобщить и сравнить невозможно.

Знай, что правда бывает другая,
А не та, что приносят на блюде.
Присмотрись к тем, чьи судьбы пугают,
Это – самые сильные люди.

Не говори, что я тебя не помню —
Я помню всё, и много раз на дню
Я повторяю номер телефонный,
Но никогда тебе не позвоню.
Вот-вот, казалось, сердце разорвется
И на пределе одиноких дней

За горизонт зашли в душе моей.
Была любовь, была любовь, была!
И к этой фразе нечего прибавить.
Сгорел волшебный замок наш дотла
И пепла не оставил нам на память.
Я помню всё, и сад цветущий помню,
И сквозь листву — лучи со всех сторон,
Как будто с белой-белой колокольни
В душе — ты слышишь — льётся тихий звон.
Любовь ушла и больше не вернётся,
И чтоб не вечно тосковать о ней,
Твои глаза, как два печальных солнца,
За горизонт зашли в душе моей.

За счастьем погоня опять неудачна…
И вечер дождливый, на улице мрачно…
А в детстве…намазала булку вареньем
И точно счастливая, до одуренья…

Гламур, этикет, бриллианты, джакузи…
Теперь, кроме счастья, в судьбе «All inclusive»,
А в детстве с подсолнуха семечки ела,
И счастью, казалось, не будет предела…

Мы стали похожи на клоунов очень…
У каждого грим, что снаружи хохочет…
А в детстве… лишь солнце с небес пробивалось
И сердце счастливое так улыбалось…

Людей отбираем, как в «Золушке» гречку…
Всех нужных – в контакты… Невыгодных в печку…
А в детстве в нас верило чистое небо…
Где радость от запаха свежего хлеба?

И дружба теперь покупается тоже…
Дожились… Живём в мире меха и кожи…
А в детстве дворнягу от ливня спасали…
И счастье давая, его получали.

Мы искренность, чуткость теряли с годами…
Границы и рамки придумали сами…
Есть булка и банка с вишнёвым вареньем?
Так будьте счастливыми, до одуренья!

Я смотрю на тебя и понимаю, что по-прежнему люблю тебя. Эта любовь – хроническая болезнь последних лет. Она приносит настолько нестерпимую боль, что я кидаюсь на совершенно посторонних людей, пытаясь обмануться ими, с ними вдруг в этих объятиях найду то самое обезболивающее, которое, по словам обладателей морщинистых сердец, вообще не существует. Я понимаю, что обманываюсь, но все равно продолжаю обниматься-убиваться не могу иначе, болит ведь, изводит, по ночам спать не дает, вот сижу на подоконнике и, еще минута, истошно закричу от пыток иллюзий. Обратиться к тебе за помощью? Бесполезно. Ты знаешь о моей любви, но тебе она ни к чему, «своих невысказанных чувств полный рот». Мы в одной паутине безответности, но не можем помочь друг другу. Ты обхватываешь руками тонкие белые нити-прутья и смотришь куда-то за пределы реальности, надеясь черт знает на чью помощь. И разница между нами одна: моя любовь к тебе почти сбила меня с ног, а твоя любовь к кому-то – подпитывает, оживляет тебя ожиданием, пусть и обманчивым. Я больше не хочу смотреть на тебя, я прогоняю возможность тебя из сердца, но от этого еще больнее. Вот и проходится шепотом страдать, тоже надеясь черт знает на чью помощь. Времени?..

Ваша жизнь – сплошное вранье, порнуха, бытовуха, интернет-зависимость и сотово-мобильное рабство. Ну разве я не прав? Вот скажите мне, вы когда-нибудь совершали что-нибудь по настоящему из ряда вон? Никогда. И не сможете. Знаете почему? Потому что все это находится за пределами вашей зоны комфорта. Вы упакованы в нее. Как в полипропиленовый мешок. Вы куски мяса, зажатые рамками быта и работы. Или я не прав? Может, я ошибаюсь? Поправьте меня.
Например, можете подарить свой мобильник первому встречному? А? Вопрос на засыпку. Можете прямо сейчас отформатировать винт на вашем компьютере? Стремно? Обосрались? А знаете, почему вы этого не сделаете? Потому что это равноценно самоубийству. Вы без этого не существуете.


Хранил в себе один секрет и был в семье примерный муж.
Всё было, вроде, как всегда: жена готовила обед…
Но приключилась вдруг беда: он взял и вспомнил про секрет.
Под шум и кислый запах щей, ворчанье суженой с утра,
Он вспомнил всё до мелочей, как будто было то вчера…
…Она сидела у окна, и мягкий чудный лунный свет
Окрасил в бледные тона её прекрасный силуэт…
Струились пряди по плечам, скользили змейками на грудь…
И он подумал сгоряча: «Женюсь на ней когда – нибудь!»
Он вспомнил всё до мелочей: изгибы линий, мягкость губ…
И жар её простых речей, и за окном огромный дуб.
Сплетенье рук… Слиянье тел… Каскад каштановых волос…
И то, как он её хотел до исступления, до слёз!
Признаний трепетных поток, как он на ушко их шептал!
Смешной над ухом завиток, что от дыханья трепетал…
Она смотрела на него глазами влажными, как ночь.
Слова пьянили, как вино: «Люблю тебя… Роди мне дочь…»
С утра он потерял покой: то суетился, то скучал…
Потом, закрыв лицо рукой, сидел на стуле и молчал.
Жена ворчала, как всегда. Ругала убежавший суп…
И он отметил, что года ей, постаревшей, не к лицу.
Как не идёт ей белый цвет и пряди крашеных волос.
И целых двадцать восемь лет всё как – то было не всерьёз…
Вдруг он вскочил, схватил пальто, забыл про шапку и носки.
Все двадцать восемь лет – не то… Все двадцать восемь зим – тоски.
Нашёл тот дом. У дома – дуб. Взбежал по лестнице стрелой…
Унять бы дрожь с холодных губ, и трусость гадкую – долой!
Наверное, она сейчас пьет чай и кутается в шаль…
И из её прекрасных глаз струится тихая печаль…
А может, принялась вязать? А может кружево плести?
Так много надо ей сказать! А главное сказать – прости…
Открыла дверь… В глазах – вопрос. Ей было снова двадцать лет…
Каскад каштановых волос… Знакомый сердцу силуэт…
Над ухом – лёгкий завиток… Как много лет назад – точь в точь…
” Вы не ошиблись?» – Нет, не мог… Вы Аня? ” Вера. Её дочь…»
” А Аня?”- ” Мамы больше нет… Кто Вы?» Он повернулся вспять:
«Я шёл к ней двадцать восемь лет…» – Она ждала Вас… Двадцать пять…
Как закружилась голова… Как сердце ухнуло в груди!
И вспомнил он её слова с мольбою: «Ты не уходи!»
Он сгорбился. Поплёлся прочь. Сплетенье рук… Слиянье тел…
Люблю тебя… Роди мне дочь… А он ведь вправду дочь хотел.
Как странно. Ани больше нет… Заплакал… Бросил в тишину:«Я буду много – много лет любить тебя… Тебя одну…»

P.S. БЕРЕГИТЕ ЛЮБОВЬ – она фундамент вашего счастья…





Пределы доставляют всем студентам, изучающим математику, немало хлопот. Чтобы решить предел, порой приходится применять массу хитростей и выбирать из множества способов решения именно тот, который подойдет для конкретного примера.

В этой статье мы не поможем вам понять пределы своих возможностей или постичь пределы контроля, но постараемся ответить на вопрос: как понять пределы в высшей математике? Понимание приходит с опытом, поэтому заодно приведем несколько подробных примеров решения пределов с пояснениями.

Понятие предела в математике

Первый вопрос: что это вообще за предел и предел чего? Можно говорить о пределах числовых последовательностей и функций. Нас интересует понятие предела функции, так как именно с ними чаще всего сталкиваются студенты. Но сначала – самое общее определение предела:

Допустим, есть некоторая переменная величина. Если эта величина в процессе изменения неограниченно приближается к определенному числу a , то a – предел этой величины.

Для определенной в некотором интервале функции f(x)=y пределом называется такое число A , к которому стремится функция при х , стремящемся к определенной точке а . Точка а принадлежит интервалу, на котором определена функция.

Звучит громоздко, но записывается очень просто:

Lim – от английского limit – предел.

Существует также геометрическое объяснение определения предела, но здесь мы не будем лезть в теорию, так как нас больше интересует практическая, нежели теоретическая сторона вопроса. Когда мы говорим, что х стремится к какому-то значению, это значит, что переменная не принимает значение числа, но бесконечно близко к нему приближается.

Приведем конкретный пример. Задача – найти предел.

Чтобы решить такой пример, подставим значение x=3 в функцию. Получим:

Кстати, если Вас интересуют , читайте отдельную статью на эту тему.

В примерах х может стремиться к любому значению. Это может быть любое число или бесконечность. Вот пример, когда х стремится к бесконечности:

Интуитивно понятно, что чем больше число в знаменателе, тем меньшее значение будет принимать функция. Так, при неограниченном росте х значение 1/х будет уменьшаться и приближаться к нулю.

Как видим, чтобы решить предел, нужно просто подставить в функцию значение, к которому стремиться х . Однако это самый простой случай. Часто нахождение предела не так очевидно. В пределах встречаются неопределенности типа 0/0 или бесконечность/бесконечность . Что делать в таких случаях? Прибегать к хитростям!


Неопределенности в пределах

Неопределенность вида бесконечность/бесконечность

Пусть есть предел:

Если мы попробуем в функцию подставить бесконечность, то получим бесконечность как в числителе, так и в знаменателе. Вообще стоит сказать, что в разрешении таких неопределенностей есть определенный элемент искусства: нужно заметить, как можно преобразовать функцию таким образом, чтобы неопределенность ушла. В нашем случае разделим числитель и знаменатель на х в старшей степени. Что получится?

Из уже рассмотренного выше примера мы знаем, что члены, содержащие в знаменателе х, будут стремиться к нулю. Тогда решение предела:

Для раскрытия неопределенностей типа бесконечность/бесконечность делим числитель и знаменатель на х в высшей степени.


Кстати! Для наших читателей сейчас действует скидка 10% на

Еще один вид неопределенностей: 0/0

Как всегда, подстановка в функцию значения х=-1 дает 0 в числителе и знаменателе. Посмотрите чуть внимательнее и Вы заметите, что в числителе у нас квадратное уравнение. Найдем корни и запишем:

Сократим и получим:

Итак, если Вы сталкиваетесь с неопределенностью типа 0/0 – раскладывайте числитель и знаменатель на множители.

Чтобы Вам было проще решать примеры, приведем таблицу с пределами некоторых функций:

Правило Лопиталя в пределах

Еще один мощный способ, позволяющий устранить неопределенности обоих типов. В чем суть метода?

Если в пределе есть неопределенность, берем производную от числителя и знаменателя до тех пор, пока неопределенность не исчезнет.

Наглядно правило Лопиталя выглядит так:

Важный момент : предел, в котором вместо числителя и знаменателя стоят производные от числителя и знаменателя, должен существовать.

А теперь – реальный пример:

Налицо типичная неопределенность 0/0 . Возьмем производные от числителя и знаменателя:

Вуаля, неопределенность устранена быстро и элегантно.

Надеемся, что Вы сможете с пользой применить эту информацию на практике и найти ответ на вопрос “как решать пределы в высшей математике”. Если нужно вычислить предел последовательности или предел функции в точке, а времени на эту работу нет от слова «совсем», обратитесь в профессиональный студенческий сервис за быстрым и подробным решением.

Матанализ с нуля. Как решать пределы для чайников? Геометрический смысл производной

Пределы доставляют всем студентам, изучающим математику, немало хлопот. Чтобы решить предел, порой приходится применять массу хитростей и выбирать из множества способов решения именно тот, который подойдет для конкретного примера.

В этой статье мы не поможем вам понять пределы своих возможностей или постичь пределы контроля, но постараемся ответить на вопрос: как понять пределы в высшей математике? Понимание приходит с опытом, поэтому заодно приведем несколько подробных примеров решения пределов с пояснениями.

Понятие предела в математике

Первый вопрос: что это вообще за предел и предел чего? Можно говорить о пределах числовых последовательностей и функций. Нас интересует понятие предела функции, так как именно с ними чаще всего сталкиваются студенты. Но сначала – самое общее определение предела:

Допустим, есть некоторая переменная величина. Если эта величина в процессе изменения неограниченно приближается к определенному числу a , то a – предел этой величины.

Для определенной в некотором интервале функции f(x)=y пределом называется такое число A , к которому стремится функция при х , стремящемся к определенной точке а . Точка а принадлежит интервалу, на котором определена функция.

Звучит громоздко, но записывается очень просто:

Lim – от английского limit – предел.

Существует также геометрическое объяснение определения предела, но здесь мы не будем лезть в теорию, так как нас больше интересует практическая, нежели теоретическая сторона вопроса. Когда мы говорим, что х стремится к какому-то значению, это значит, что переменная не принимает значение числа, но бесконечно близко к нему приближается.

Приведем конкретный пример. Задача – найти предел.

Чтобы решить такой пример, подставим значение x=3 в функцию. Получим:

Кстати, если Вас интересуют базовые операции над матрицами , читайте отдельную статью на эту тему.

В примерах х может стремиться к любому значению. Это может быть любое число или бесконечность. Вот пример, когда х стремится к бесконечности:

Интуитивно понятно, что чем больше число в знаменателе, тем меньшее значение будет принимать функция. Так, при неограниченном росте х значение 1/х будет уменьшаться и приближаться к нулю.

Как видим, чтобы решить предел, нужно просто подставить в функцию значение, к которому стремиться х . Однако это самый простой случай. Часто нахождение предела не так очевидно. В пределах встречаются неопределенности типа 0/0 или бесконечность/бесконечность . Что делать в таких случаях? Прибегать к хитростям!


Неопределенности в пределах

Неопределенность вида бесконечность/бесконечность

Пусть есть предел:

Если мы попробуем в функцию подставить бесконечность, то получим бесконечность как в числителе, так и в знаменателе. Вообще стоит сказать, что в разрешении таких неопределенностей есть определенный элемент искусства: нужно заметить, как можно преобразовать функцию таким образом, чтобы неопределенность ушла. В нашем случае разделим числитель и знаменатель на х в старшей степени. Что получится?

Из уже рассмотренного выше примера мы знаем, что члены, содержащие в знаменателе х, будут стремиться к нулю. Тогда решение предела:

Для раскрытия неопределенностей типа бесконечность/бесконечность делим числитель и знаменатель на х в высшей степени.


Кстати! Для наших читателей сейчас действует скидка 10% на любой вид работы

Еще один вид неопределенностей: 0/0

Как всегда, подстановка в функцию значения х=-1 дает 0 в числителе и знаменателе. Посмотрите чуть внимательнее и Вы заметите, что в числителе у нас квадратное уравнение. Найдем корни и запишем:

Сократим и получим:

Итак, если Вы сталкиваетесь с неопределенностью типа 0/0 – раскладывайте числитель и знаменатель на множители.

Чтобы Вам было проще решать примеры, приведем таблицу с пределами некоторых функций:

Правило Лопиталя в пределах

Еще один мощный способ, позволяющий устранить неопределенности обоих типов. В чем суть метода?

Если в пределе есть неопределенность, берем производную от числителя и знаменателя до тех пор, пока неопределенность не исчезнет.

Наглядно правило Лопиталя выглядит так:

Важный момент : предел, в котором вместо числителя и знаменателя стоят производные от числителя и знаменателя, должен существовать.

А теперь – реальный пример:

Налицо типичная неопределенность 0/0 . Возьмем производные от числителя и знаменателя:

Вуаля, неопределенность устранена быстро и элегантно.

Надеемся, что Вы сможете с пользой применить эту информацию на практике и найти ответ на вопрос “как решать пределы в высшей математике”. Если нужно вычислить предел последовательности или предел функции в точке, а времени на эту работу нет от слова «совсем», обратитесь в профессиональный студенческий сервис за быстрым и подробным решением.

В категории Математический анализ собраны бесплатные онлайн видео уроки по этой теме. Математический анализ – это совокупность разделов математики, которые занимаются изучением функций и их обобщений методами дифференциального и интегрального исчисления. Сюда относятся: функциональный анализ, включая теорию интеграла Лебега, комплексный анализ (ТФКП), изучающий функции, заданные на комплексной плоскости, теория рядов и многомерных интегралов, нестандартный анализ, изучающий бесконечно малые и бесконечно большие числа, векторный анализ, а также вариационное исчисление. Изучение математического анализа по видео урокам будет полезно как для начинающих, так и для более опытных математиков. Видеоуроки из рубрики Математический анализ Вы можете смотреть бесплатно в любое удобное время. К некоторым видео урокам по математическому анализу приложены дополнительные материалы, которые можно скачать. Приятного Вам обучения!

Всего материалов: 12
Показано материалов: 1-10

Что такое производная функции

Хотите узнать, что такое производная функции в математике? Ты конечно много раз слышал о производной и даже, наверное, брал эту самую производную в школе, совершенно не понимая смысла своих действий. В этом видео я не буду учить тебя формулам, а объясню смысл производной на пальцах так, чтобы даже круглому чайнику было понятно. Но сначала тебе лучше посмотреть мое предыдущее видео, где я так же доступно рассказываю о функции. В этом видеоуроке мы просто, понятно и наглядных жизненных примерах…

Введение в анализ. Мощность множеств

Онлайн урок «Введение в анализ. Мощность множеств» посвящен вопросу о таком понятии как мощность множеств. Этот вопрос касается количественной характеристики множеств. Если множество конечно, то можно говорить о количестве его элементов. Но как быть с бесконечными множествами? Ведь в этом случае не будет понятия больше или меньше. Для решения этой задачи вводится такое понятие как мощность. Мощность – это инструмент количественного сравнения бесконечных множеств. В этом занятии дается…

Предел функции в точке – определение, примеры

В этом онлайн уроке рассказывается о таком понятии как предел функции в точке – определение, примеры. Большинство элементов исследования функций опираются на базовое понятие предела функции. Здесь будет рассмотрен предел функции в точке на простом примере, после чего будет дано строгое определение предела функции в точке с подробной иллюстрацией на графике для лучшего усвоения материала. На данном занятии также рассматриваются другие примеры, и сформулировано строгое определение односторонних…

Сходимость степенных рядов – пример как найти область сходимости, исследование

В этом видео уроке рассказывается о таком понятии как сходимость степенных рядов, пример как найти область сходимости, исследование. Степенной ряд – это частный случай функционального ряда, когда его членами являются степенные функции аргумента x. Область сходимости представляет собой все значения переменной x, при которых соответствующие числовые ряды сходятся. Для исследования можно использовать признак Даламбера и с помощью него показать, что степенной ряд сходится или расходится, и при…

Что такое первообразная

В этом видео я расскажу тебе про первообразную, которая является близкой родственницей производной. На самом деле ты про нее и так уже почти все знаешь, если посмотрел мои предыдущие видео, и нам осталось только расставить точки над i. Первообразная – это «родительская» функция для производной. Найти первообразную – это значит ответить на вопрос: а чей ребенок? Если известна дочка, то мы должны найти маму. Раньше мы наоборот искали дочку по заданной маме. Сейчас мы делаем обратный переход – от…

Геометрический смысл производной

В этом видео я расскажу про геометрический смысл производной. Ты узнаешь, что геометрический смысл производной состоит в том, что производная и угол наклона касательной – это почти одно и то же. Я говорю «почти» потому что производная равна тангенсу угла наклона касательной. Можно считать, что производная и наклон касательной – они тесно связаны. Если угол наклона большой, то и производная большая, а функция в этой точке резко возрастает. Если угол наклона маленький, то и производная маленькая…

Что такое функция в математике

Хотите узнать, что такое функция в математике? В этом видеоуроке мы просто и понятно, с использованием графических иллюстраций и на наглядных жизненных примерах расскажем, что такое функция, что такое ее аргумент, какие бывают функции (возрастающие, убывающие, смешанные), как можно задать функцию (с помощью графика, таблицы, формулы). Вы увидите, что зависимость, которая показывает, как одна величина связана с другой величиной, называется функцией. Любая функция – это связь между величинами…

Предел функции на бесконечности – определение, примеры

Урок «Предел функции на бесконечности – определение, примеры» посвящен вопросу о том, что такое пределы на бесконечности. Большинство элементарных функций определено для сколь угодно большого значения аргумента. В этом случае важно знать поведение функции на бесконечности. Один из элементов исследования такого поведения является нахождение предела функции на бесконечности. Хотя бесконечность не является числом, и ей не соответствует ни одна точка на числовой прямой, определение предела на…

Для тех, кто хочет научиться находить пределы в данной статье мы расскажем об этом. Не будем углубляться в теорию, обычно её дают на лекциях преподаватели. Так что “скучная теория” должна быть у Вас законспектирована в тетрадках. Если этого нет, то почитать можно учебники взятые в библиотеке учебного заведения или на других интернет-ресурсах.

Итак, понятие предела достаточно важно в изучении курса высшей математики, особенно когда вы столкнетесь с интегральным исчислением и поймёте связь между пределом и интегралом. В текущем материале будут рассмотрены простые примеры, а также способы их решения.

Примеры решений

New Page 1

Понятие множества

Множество – это совокупность некоторых объектов. Какие могут быть множества? Во первых, конечные или бесконечные. Например, множество спичек в коробке – это конечное множество, их можно взять и сосчитать. Количество песчинок на пляже сосчитать гораздо труднее, но, в принципе, возможно. И это количество выражается каким то конечным числом. Так что множество песчинок на пляже тоже конечно. А вот множество точек на прямо это множество бесконечное. Так как во первых, прямая сама по себе бесконечная и на ней можно поставить сколько угодно точек. Множество точек отрезка прямой тоже бесконечное. Потому что теоретически точка может быть сколь угодно маленькая. Конечно, мы физически не сможем нарисовать точку, размером, например, меньше размера атома, но, с точки зрения математики точка не имеет размера. Ее размер равен нулю. А что получается, если разделить на нуль какое то число? Правильно, бесконечность. И хотя множество точек на прямой и на отрезке стремится к бесконечности, это не одно и тоже. Множество – это не количество чего то там, а совокупность каких либо объектов. И равными считаются только те множества, которые содержат абсолютно одинаковые объекты. Если в одном множество содержит те же объекты, что и другое множество, но плюс еще один какой нибудь “левый” объект, то это уже не равные множества.

Рассмотрим пример. Пусть у нас имеется два множества. Первое – совокупность все точек на прямой. Второе – совокупность всех точек на отрезке прямой. Почему они не равны? Во первых, отрезок и прямая могут даже не пересекаться. Тогда они уж точно не равны, так как содержат в себе абсолютно разные точки. Если они пересекаются, то у них только одна общая точка. Все остальные так же разные. А если отрезок лежит на прямой? Тогда все точки отрезка являются и точками прямой. Но не все точки прямой являются точками отрезка. Так что и в этом случае множества нельзя считать равными (одинаковыми).

Каждое множество задается правилом, которое однозначно определяет, принадлежит элемент к этому множеству или нет. Какие могут быть эти правила? Например, если множество конечное, можно тупо перечислить все его объекты. Можно задать диапазон. Например, все целые числа от 1 до 10. Это будет тоже конечное множество, но тут мы не перечисляем его элементы, а формулируем правило. Или неравенство, к примеру, все числа, больше 10. Это будет уже бесконечное множество, поскольку нельзя назвать самое большое число – какие бы число мы не называли, всегда есть это число плюс 1.

Как правило, множества обозначаются прописными буквами латинского алфавита A, B, C и так далее. Если множество состоит из конкретных элементов и мы хотим задать его списком этих элементов, то мы можем заключить этот список в фигурные скобки, например A={a, b, c, d}. Если a является элемент множества A, то это записывают следующим образом: a Î A . Если же a не является элементом множества A, то пишут a Ï A. Одним из важных множеств является множество N всех натуральных чисел N={1,2,3,…,} . Существует также специальное, так называемое пустое множество, которое не содержит ни одного элемента. Пустое множество обозначается символом Æ .

Определение 1 (определение равенства множеств). Множества А и B равны, если они состоят из одних и тех же элементов, то есть, если из x Î A следует x Î B и обратно, из x Î B следует x Î A.

Формально равенство двух множеств записывается следующим образом:

(А=В ) := x (( x Î A ) Û (x Î B )),

Это означает, что для любого объекта x соотношения x Î A и x Î B равносильны.

Здесь ” – квантор всеобщности (” x читается как “для каждого x “).

Определение 2 (определение подмножества). Множество А является подмножеством множества В , если любое х принадлежащее множеству А , принадлежит множеству В. Формальное это можно представить в виде выражения:

(A Ì B ) := x ((x Î A ) Þ (x Î B ))

Если A Ì B, но A ¹ B, то A – собственное подмножество множества В. В качестве примера можно привести опять же прямую и отрезок. Если отрезок лежит на прямой, то множество его точек являются подмножеством точек этой прямой. Или, другой пример. Множество целых чисел, которые делятся без остатка на 3, является подмножеством множества целых чисел.

Замечание. Пустое множество является подмножеством любого множества.

Операции над множествами

Над множествами возможны следующие операции:

Объединение. Суть этой операции состоит в том, что бы два множества объединить в одно, содержащее элементы каждого из объединяемых множеств. Формально это выглядит так:

C=A È B: = {x:x Î A или x Î B }

Пример. Решим неравенство | 2 x + 3 | > 7.

Из него следует либо неравенство 2x+3 >7, для 2x+3 ≥0, тогда x>2

либо неравенство 2x+3

Множеством решений данного неравенство является объединения множеств (-∞,-5) È (2, ∞).

Давайте проверим. Посчитаем значение выражение | 2 x + 3 | для нескольких точек, лежащих и не лежащих в данном диапазоне:

x | 2 x + 3 |
-1017
-69
-57
-45
-21
03
15
27
39
513

Как видим, все решено правильно (красным обозначены пограничные диапазоны).

Пересечение. Пересечением называется операция создания нового множества из двух, содержащих элементы, которые входят в оба этих множества. Что бы изобразить это наглядно, давайте представим, что у нас есть два множества точек на плоскости, а именно фигура A и фигура B. Их пересечение обозначает фигуру C – это и есть результа операции пересечения множеств:

Формально операция пересечения множеств записывается так:

C=A Ç B := {x: x Î A и x Î B }

Пример. Пусть у нас есть множество Тогда

C=A Ç B = {5,6,7}

Вычитание. Вычитание множеств – это исключение из вычитаемого множества тех элементах, которые содержатся в вычитаемом и вычитателе:

Формально вычитание множества записывается так:

A \ B: = {x:x Î A и x Ï B }

Пример. Пусть у нас есть множество A={1,2,3,4,5,6,7}, B={5,6,7,8,9,10}. Тогда C=A \ B = { 1,2,3,4}

Дополнение. Дополнение – это унарная операция (операция не над двумя, а над одним множеством). Эта операция является результатом вычитания данного множества из полного универсального множества (множества, которое включает в себя все остальные множества).

A : = {x:x Î U и x Ï A} = U \ A

Графически это можно изобразить в виде:

Симметричная разность. В отличии от обычной разности при симметричной разности множеств элементы остаются только те, что присутствуют либо в одном, либо в другом множестве. Или, говоря простым языком, из двух множеств создается, но из него исключаются те элементы, которые есть и в том и в другом множестве:

Математически это можно выразить так:

A D B:= (A \ B ) È (B \ A ) = (A È B ) \ (A Ç B )

Свойства операций над множествами.

Из определений объединения и пересечения множеств следует, что операции пересечения и объединения обладают следующими свойствами:

  1. Коммутативность.

A È B=B È A
A
Ç B=B Ç A

  1. Ассоциативность.

(A È B ) È C=A È (B È C )
(A Ç B ) Ç C= A Ç (B Ç C )

Пример 1
Вычислить а) $ \lim_{x \to 0} \frac{1}{x} $; б)$ \lim_{x \to \infty} \frac{1}{x} $
Решение

а) $$ \lim \limits_{x \to 0} \frac{1}{x} = \infty $$

б)$$ \lim_{x \to \infty} \frac{1}{x} = 0 $$

Нам часто присылают эти пределы с просьбой помочь решить. 2-1}{x+1} = \infty $$

Алгоритм вычисления лимитов

Итак, давайте кратко подведем итог разобранным примерам и составим алгоритм решения пределов:

  1. Подставить точку х в выражение, следующее после знака предела. Если получается определенное число, либо бесконечность, то предел решен полностью. В противном случае имеем неопределенность: “ноль делить на ноль” или “бесконечность делить на бесконечность” и переходим к следующим пунктам инструкции.
  2. Чтобы устранить неопределенность “ноль делить на ноль” нужно разложить числитель и знаменатель на множители. Сократить подобные. Подставить точку х в выражение, стоящее под знаком предела.
  3. Если неопределенность “бесконечность делить на бесконечность”, тогда выносим и в числителе, и в знаменателе x наибольшей степени. Сокращаем иксы. Подставляем значения икса из под предела в оставшееся выражение.

В этой статье Вы ознакомились с основами решения пределов, часто используемых в курсе Математического анализа. Конечно же это не все типы задач, предлагающихся экзаменаторами, а только простейшие пределы. В следующих статьях поговорим о других типах заданий, но сперва необходимо усвоить этот урок, чтобы двигаться далее. Обсудим, что делать, если есть корни, степени, изучим бесконечно малые эквивалентные функции, замечательные пределы, правило Лопиталя.

Если у Вас не получается самостоятельно решить пределы, то не паникуйте. Мы всегда рады помочь!

Матанализ для чайников – матан с примерами и решениями

Содержание:

  1. Замечания о доказательствах
  2. Некоторые специальные обозначения
  3. Заключительные замечания
  4. Понятие множества
  5. Понятие функции (отображения)
  6. Пример решения
  7. Целые числа
  8. Рациональные числа
  9. Иррациональные числа
  10. Предел. Определения и примеры
  11. Примеры с решением
  12. Критерий Коши
  13. Пример с решением

Язык этой страницы, как и большинства математических текстов, состоит из обычного языка и ряда специальных символов излагаемых теорий. Наряду с этими специальными символами, которые будут вводиться по мере надобности, мы используем распространенные символы математической логики для обозначения соответственно отрицания «не» и связок «и», «или», «влечет», «равносильно».

Возьмем, например, три представляющих и самостоятельный интерес высказывания:

L. «Если обозначения удобны для открытий …, то поразительным образом сокращается работа мысли» (Г. Лейбниц)).

Р. «Математика — это искусство называть разные вещи одинаковыми именами» (А. Пуанкаре)).

G. «Великая книга природы написана языком математики» (Г. Галилей)).

Тогда в соответствии с указанными обозначениями:

Мы видим, что пользоваться только формальными обозначениями, избегая разговорного языка, — не всегда разумно.

Мы замечаем, кроме того, что в записи сложных высказываний, составленных из более простых, употребляются скобки, выполняющие ту же синтаксическую функцию, что и при записи алгебраических выражений.

По этой ссылке вы найдёте полный курс лекций по высшей математике:

Как и в алгебре, для экономии скобок можно договориться о «порядке действий». Условимся с этой целью о следующем порядке приоритета символов:

При таком соглашении выражение следует расшифровать как a соотношение но не как

Записи означающей, что влечет или, что то же самое, следует из , мы часто будем придавать другую словесную интерпретацию, говоря, что есть необходимый признак или необходимое условие и, в свою очередь, — достаточное условие или достаточный признак Таким образом, соотношение можно прочитать любым из следующих способов:

необходимо и достаточно для

тогда и только тогда, когда

, если и только если

равносильно

Итак, запись означает, что влечет и, одновременно, влечет .

Употребление союза в выражении пояснений не требует.

Возможно вам будут полезны данные страницы:

Следует, однако, обратить внимание на то, что в выражении союз или неразделительный, т. е. высказывание считается верным, если истинно хотя бы одно из высказываний Например, пусть — такое действительное число, что Тогда можно написать, что имеет место следующее соотношение:

Замечания о доказательствах

Типичное математическое утверждение имеет вид где — посылка, а — заключение. Доказательство такого утверждения состоит в построении цепочки следствий, каждый элемент которой либо считается аксиомой, либо является уже доказанным утверждением).

В доказательствах мы будем придерживаться классического правила вывода: если истинно и то тоже истинно.

При доказательстве от противного мы будем использовать также принцип исключенного третьего, в силу которого высказывание считается истинным независимо от конкретного содержания высказывания Следовательно, мы одновременно принимаем, что т. е. повторное отрицание равносильно исходному высказыванию.

Некоторые специальные обозначения

Для удобства читателя и сокращения текста начало и конец доказательства условимся отмечать знаками и соответственно.

Условимся также, когда это будет удобно, вводить определения посредством специального символа (равенство по определению), в котором двоеточие ставится со стороны определяемого объекта.

Например, запись

определяет левую часть посредством правой части, смысл которой предполагается известным.

Аналогично вводятся сокращенные обозначения для уже определенных выражений. Например,запись

вводит обозначение для стоящей слева суммы специального вида.

Заключительные замечания

Отметим, что мы здесь говорили, по существу, только об обозначениях, не анализируя формализм логических выводов и не касаясь глубоких вопросов истинности, доказуемости, выводимости, составляющих предмет исследования математической логики.

Как же строить математический анализ, если мы не имеем формализации логики? Некоторое утешение тут может состоять в том, что мы всегда знаем или, лучше сказать, умеем больше, чем способны в данный момент формализовать. Пояснением смысла последней фразы может служить известная притча о том, что сороконожка даже ходить разучилась, когда ее попросили объяснить, как именно она управляется со всеми своими конечностями.

Опыт всех наук убеждает нас в том, что считавшееся ясным или простым и нерасчленяемым вчера может подвергнуться пересмотру или уточнению сегодня. Так было (и, без сомнения, еще будет) и с многими понятиями математического анализа, важнейшие теоремы и аппарат которого были открыты еще в XVII —XVIII веках, но приобрели современный формализованный, однозначно трактуемый и, вероятно, потому общедоступный вид лишь после создания теории пределов и необходимой для нее логически полноценной теории действительных чисел (XIX век).

Именно с этого уровня теории действительных чисел мы и начнем в главе II построение всего здания анализа

Как уже отмечалось в предисловии, желающие быстрее ознакомиться с основными понятиями и эффективным аппаратом собственно дифференциального и интегрального исчисления могут начать сразу с III главы, возвращаясь к отдельным местам первых двух глав лишь по мере необходимости.

Понятие множества

С конца прошлого — начала нашего столетия наиболее универсальным языком математики стал язык теории множеств. Это проявилось даже в одном из определений математики как науки, изучающей различные структуры (отношения) на множествах).

«Под множеством мы понимаем объединение в одно целое определенных, вполне различимых объектов нашей интуиции или нашей мысли» — так описал понятие «множество» Георг Кантор, основатель теории множеств.

Описание Кантора, разумеется, нельзя назвать определением, поскольку оно апеллирует к понятиям, быть может, более сложным (во всяком случае, не определенным ранее), чем само понятие множества. Цель этого описания — разъяснить понятие, связав его с другими.

Основные предпосылки канторовской (или, как условно говорят, «наивной») теории множеств сводятся к следующему:

  • 1 Множество может состоять из любых различимых объектов.
  • 2 Множество однозначно определяется набором составляющих его объектов.
  • 3 Любое свойство определяет множество объектов, которые этим свойством обладают.

Если — объект, — свойство, — обозначение того, что обладает свойством то через обозначают весь класс объектов, обладающих свойством Объекты, составляющие класс или множество, называют элементами класса или множества.

Понятие функции (отображения)

Перейдем теперь к описанию фундаментального не только для математики понятия функциональной зависимости.

Пусть — какие-то множества.

Говорят, что имеется функция, определенная на со значениями в если в силу некоторого закона каждому элементу соответствует элемент

В этом случае множество называется областью определения функции; символ его общего элемента — аргументом функции или независимой переменной; соответствующий конкретному значению аргумента элемент называют значением функции на элементе или значением функции при значении аргумента и обозначают через При изменении аргумента значения вообще говоря, меняются в зависимости от значений По этой причине величину часто называют зависимой переменной.

Множество

всех значений функции, которые она принимает на элементах множества будем называть множеством значений или областью-значений функции.

В зависимости от природы множеств термин «функция» в различных отделах математики имеет ряд полезных синонимов: отображение, преобразование, морфизм, оператор, функционал. Отображение — наиболее распространенный из них, и мы его тоже часто будем употреблять.

Для функции (отображения) приняты следующие обозначения:

Пример решения

Условие:

Формулы устанавливают функциональную зависимость длины окружности и объема шара от радиуса По смыслу каждая из этих формул задает свою функцию определенную на множестве положительных действительных чисел со значениями в том же множестве

Решение:

Пусть — множество инерциальных систем координат, а — функция, состоящая в том, что каждой инерциальной системе координат сопоставляется измеренное относительно нее значение скорости света в вакууме. Функция постоянна, т. е. при любом она имеет одно и то же значение (это фундаментальный экспериментальный факт).

Целые числа

Определение 3. Объединение множества натуральных чисел, множества чисел, противоположных натуральным числам, и нуля называется множеством целых чисел и обозначается символом

Поскольку, как уже было доказано, сложение и умножение натуральных чисел не выводят за пределы то эти же операции над целыми числами не выводят за пределы множества

Действительно, если то либо одно из этих чисел равно нулю и тогда сумма равна другому числу, т. е. a произведение либо оба числа отличны от нуля. В последнем случае либо и тогда либо и тогда либо и тогда т.е. либо, наконец, и тогда и снова

Таким образом, есть абелева группа по отношению к операции сложения. По отношению к операции умножения множество и даже не является группой, поскольку числа, обратные целым, не лежат в (кроме числа, обратного единице и минус единице).

Рациональные числа

Определение 4. Числа вида называются рациональными.

Множество рациональных чисел обозначается знаком Таким образом, упорядоченная пара целых чисел определяет рациональное число если

Число записывают также в виде отношения или так называемой рациональной дроби

Правила действий с рациональными числами, относящиеся к такой форме их представления дробями, изучавшиеся в школе, немедленно вытекают из определения рационального числа и аксиом действительных чисел. В частности, «от умножения числителя и знаменателя дроби на одно и то же отличное от нуля целое число величина дроби не изменяется», т. е. дроби и — представляют одно и то же рациональное число. В самом деле, поскольку

Иррациональные числа

Определение 5. Действительные числа, не являющиеся рациональными, называются иррациональными.

Классическим примером иррационального действительного числа является т. е. число такое, что Иррациональность в силу теоремы Пифагора эквивалентна утверждению о несоизмеримости диагонали и стороны квадрата.

Итак, проверим, во-первых, что существует положительное действительное число квадрат которого равен двум, и, во-вторых, что

Пусть — множества положительных действительных чисел такие, что Поскольку -непустые множества.

Далее, поскольку для положительных то любой элемент меньше любого элемента По аксиоме полноты существует число такое, что

Покажем, что

Если бы было то, например, квадрат числа большего чем был бы меньше 2. Действительно, ведь поэтому и Значит,

Следовательно, что несовместимо с неравенством для любого элемента

Если бы было то, например, квадрат числа меньшего чем был бы больше 2. Действительно, ведь поэтому или Отсюда

и мы вступаем в противоречие с тем, что ограничивает множество снизу. Таким образом, реализуется только одна оставшаяся возможность: Покажем, наконец, что Предположим, что и пусть несократимое представление Тогда следовательно, а значит, и делится на 2. Но если и по той же причине должно делиться на 2, что противоречит несократимости дроби

Предел. Определения и примеры

Напомним следующее

Определение 1. Функция областью определения которой является множество натуральных чисел, называется последовательностью.

Значения функции называются членами последовательности. Их принято обозначать символом элемента того множества, в которое идет отображение, наделяя символ соответствующим индексом аргумента, Саму последовательность в связи с этим обозначают символом а также записывают в виде и называют последовательностью в или последовательностью элементов множества

Элемент называется членом последовательности.

Всюду дальше в ближайших параграфах будут рассматриваться только последовательности действительных чисел.

Определение 2. Число называется пределом числовой последовательности если для любой окрестности точки существует такой номер (выбираемый в зависимости от что все члены последовательности, номера которых больше содержатся в указанной окрестности точки

Ниже мы приведем формально-логическую запись этого определения, но прежде укажем другую распространенную формулировку определения предела числовой последовательности:

Число называется пределом последовательности если для любого существует номер такой, что при всех имеем

Эквивалентность этих формулировок легко проверить (проверьте!), если заметить, что в любой окрестности точки содержится некоторая окрестность этой же точки.

Последняя формулировка определения предела означает, что, какую бы точность мы ни задали, найдется номер такой, что абсолютная погрешность приближения числа членами последовательности меньше чем как только

Запишем теперь приведенные формулировки определения предела в логической символике, договорившись, что запись означает, что предел последовательности Итак,

и соответственно

Определение 3. Если то говорят, что последовательность сходится к или стремится к и пишут при

Последовательность, имеющая предел, называется сходящейся. Последовательность, не имеющая предела, называется расходящейся.

Примеры с решением

Пример 1.

Пример 2.

Пример 3.

Пример 4.

Пример 5.

Проверим это по определению предела. Как было доказано в гл. И, § 2, 4с, для любого можно найти число такое, что Поскольку то для любого будем иметь и определение предела удовлетворено.

Критерий Коши

Определение 7. Последовательность называется фундаментальной (или последовательностью Кошиесли для любого числа найдется такой номер что из следует

Теорема 4 (критерий Коши сходимости последовательности). Числовая последовательность сходится тогда и только тогда, когда она фундаментальна.

Пусть По числу найдем номер так, чтобы при иметь Если теперь и, таким образом, проверено, что сходящаяся

последовательность фундаментальна.

Пусть теперь — фундаментальная последовательность. По заданному найдем номер такой, что из следует

Фиксировав получаем, что при любом

но поскольку имеется всего конечное число членов последовательности с номерами,-не превосходящими то мы доказали, что фундаментальная последовательность ограничена.

Для положим теперь

Из этих определений видно, что (поскольку при переходе к меньшему множеству нижняя грань не уменьшается, а верхняя не увеличивается). Последовательность вложенных отрезков имеет, по лемме о вложенных отрезках, общую точку

Поскольку при любом

а при

то при имеем

Но из (1) следует, что при

поэтому при

Сравнивая (2) и (3), находим, что при любом

и мы показали, что

Пример с решением

Последовательность не имеет предела, поскольку она не является фундаментальной. Хотя это и очевидно, но все же проведем формальную проверку. Отрицание утверждения, что последовательность фундаментальная, выглядит так:

т. е. найдется такое, что при любом найдутся числа большие для которых

В нашем случае достаточно положить Тогда при любом будем иметь

Числа π и e

Все знают геометрический смысл числа π — это длина окружности с единичным диаметром:

А вот смысл другой важной константы, e, имеет свойство быстро забываться. То есть, не знаю, как вам, а мне каждый раз стоит усилий вспомнить, чем же так замечательно это число, равное 2,7182818284590… (значение я, однако, по памяти записал). Поэтому я решил написать заметку, чтобы больше из памяти не вылетало.

Число e по определению — предел функции y = (1 + 1 / x)x при x → ∞:

xy
1(1 + 1 / 1)1= 2
2(1 + 1 / 2)2= 2,25
3(1 + 1 / 3)3= 2,3703703702…
10(1 + 1 / 10)10= 2,5937424601…
100(1 + 1 / 100)100= 2,7048138294…
1000(1 + 1 / 1000)1000= 2,7169239322…
lim× → ∞= 2,7182818284590. ..

Это определение, к сожалению, не наглядно. Непонятно, чем замечателен этот предел (несмотря на то, что он называется «вторым замечательным»). Подумаешь, взяли какую-то неуклюжую функцию, посчитали предел. У другой функции другой будет.

Но число e почему-то всплывает в целой куче самых разных ситуаций в математике.

Для меня главный смысл числа e раскрывается в поведении другой, куда более интересной функции, y = kx. Эта функция обладает уникальным свойством при k = e, которое можно показать графически так:

В точке 0 функция принимает значение e0 = 1. Если провести касательную в точке x = 0, то она пройдёт к оси абсцисс под углом с тангенсом 1 (в жёлтом треугольнике отношение противолежащего катета 1 к прилежащему 1 равно 1). В точке 1 функция принимает значение e1 = e. Если провести касательную в точке x = 1, то она пройдёт под углом с тангенсом e (в зелёном треугольнике отношение противолежащего катета e к прилежащему 1 равно e). В точке 2 значение e2 функции снова совпадает с тангенсом угла наклона касательной к ней. Из-за этого, заодно, сами касательные пересекают ось абсцисс ровно в точках −1, 0, 1, 2 и т. д.

Среди всех функций y = kx (например, 2x, 10x, πx и т. д.), функция ex — единственная обладает такой красотой, что тангенс угла её наклона в каждой её точке совпадает со значением самой функции. Значит по определению значение этой функции в каждой точке совпадает со значением её производной в этой точке: (ex)´ = ex. Почему-то именно число e = 2,7182818284590… нужно возводить в разные степени, чтобы получилась такая картинка.

Именно в этом, на мой вкус, состоит его смысл.

Числа π и e входят в мою любимую формулу — формулу Эйлера, которая связывает 5 самых главных констант — ноль, единицу, мнимую единицу i и, собственно, числа π и е:

e + 1 = 0

Почему число 2,7182818284590. .. в комплексной степени 3,1415926535…i вдруг равно минус единице? Ответ на этот вопрос выходит за рамки заметки и мог бы составить содержание небольшой книги, которая потребует некоторого начального понимания тригонометрии, пределов и рядов.

Меня всегда поражала красота этой формулы. Возможно, в математике есть и более удивительные факты, но для моего уровня (тройка в физико-математическом лицее и пятёрка за комплексный анализ в универе) это самое главное чудо.

Бесконечно большие и бесконечно малые функции (Лекция №2)

БЕСКОНЕЧНО МАЛЫЕ ФУНКЦИИ И ИХ ОСНОВНЫЕ СВОЙСТВА

Функция y=f(x) называется бесконечно малой при x→a или при x→∞, если или , т.е. бесконечно малая функция – это функция, предел которой в данной точке равен нулю.

Примеры.

  1. Функция f(x)=(x-1)2 является бесконечно малой при x→1, так как (см. рис.).
  2. Функция f(x) = tgx – бесконечно малая при x→0.
  3. f(x) = ln (1+x)– бесконечно малая при x→0.
  4. f(x) = 1/x– бесконечно малая при x→∞.

Установим следующее важное соотношение:

Теорема. Если функция y=f(x) представима при x→aв виде суммы постоянного числа b и бесконечно малой величины α(x): f (x)=b+ α(x) то .

Обратно, если , то f (x)=b+α(x), где a(x) – бесконечно малая при x→a.

Доказательство.

  1. Докажем первую часть утверждения. Из равенства f(x)=b+α(x) следует |f(x) – b|=| α|. Но так как a(x) – бесконечно малая, то при произвольном ε найдется δ – окрестность точки a, при всех x из которой, значения a(x) удовлетворяют соотношению |α(x)|<ε. Тогда |f(x) – b|< ε. А это и значит, что .
  2. Если , то при любом ε>0 для всех х из некоторой δ – окрестность точки a будет |f(x) – b|< ε. Но если обозначимf(x) – b= α, то |α(x)|<ε, а это значит, что a – бесконечно малая.

Рассмотрим основные свойства бесконечно малых функций.

Теорема 1. Алгебраическая сумма двух, трех и вообще любого конечного числа бесконечно малых есть функция бесконечно малая.

Доказательство. Приведем доказательство для двух слагаемых. Пусть f(x)=α(x)+β(x), где и . Нам нужно доказать, что при произвольном как угодно малом ε>0 найдется δ>0, такое, что для x, удовлетворяющих неравенству |x – a|<δ, выполняется |f(x)|< ε.

Итак, зафиксируем произвольное число ε>0. Так как по условию теоремы α(x) – бесконечно малая функция, то найдется такое δ1>0, что при |x – a|<δ1 имеем |α(x)|< ε/2. Аналогично, так как β(x) – бесконечно малая, то найдется такое δ2>0, что при |x – a|<δ2 имеем | β(x)|< ε/2.

Возьмем δ=min{ δ1, δ2}.Тогда в окрестности точки a радиуса δбудет выполняться каждое из неравенств |α(x)|< ε/2 и | β(x)|< ε/2. Следовательно, в этой окрестности будет

|f(x)|=| α(x)+β(x)| ≤ |α(x)| + | β(x)| < ε/2 + ε/2= ε,

т.е. |f(x)|<ε, что и требовалось доказать.

Теорема 2. Произведение бесконечно малой функции a(x) на ограниченную функцию f(x) при x→a (или при x→∞) есть бесконечно малая функция.

Доказательство. Так как функция f(x) ограничена, то существует число М такое, что при всех значениях x из некоторой окрестности точки a|f(x)|≤M. Кроме того, так как a(x) – бесконечно малая функция при x→a, то для произвольного ε>0 найдется окрестность точки a, в которой будет выполняться неравенство |α(x)|< ε/M. Тогда в меньшей из этих окрестностей имеем | αf|< ε/M= ε. А это и значит, что af – бесконечно малая. Для случая x→∞ доказательство проводится аналогично.

Из доказанной теоремы вытекают:

Следствие 1. Если и , то .

Следствие 2. Если и c=const, то .

Теорема 3. Отношение бесконечно малой функции α(x) на функцию f(x), предел которой отличен от нуля, есть бесконечно малая функция.

Доказательство. Пусть . Тогда 1/f(x) есть ограниченная функция. Поэтому дробь есть произведение бесконечно малой функции на функцию ограниченную, т.е. функция бесконечно малая.

СООТНОШЕНИЕ МЕЖДУ БЕСКОНЕЧНО МАЛЫМИ И БЕСКОНЕЧНО БОЛЬШИМИ ФУНКЦИЯМИ

Теорема 1. Если функция f(x) является бесконечно большой при x→a, то функция 1/f(x) является бесконечно малой при x→a.

Доказательство. Возьмем произвольное число ε>0 и покажем, что при некотором δ>0 (зависящим от ε) при всех x, для которых |x – a|<δ, выполняется неравенство , а это и будет означать, что 1/f(x) – бесконечно малая функция. Действительно, так как f(x) – бесконечно большая функция при x→a, то найдется δ>0 такое, что как только |x – a|<δ, так |f(x)|>1/ ε. Но тогда для тех же x.

Примеры.

  1. Ясно, что при x→+∞ функция y=x2+1 является бесконечно большой. Но тогда согласно сформулированной выше теореме функция – бесконечно малая при x→+∞, т.е. .
  2. .

Можно доказать и обратную теорему.

Теорема 2. Если функция f(x) - бесконечно малая при x→a (или x→∞) и не обращается в нуль, то y=1/f(x) является бесконечно большой функцией.

Доказательство теоремы проведите самостоятельно.

Примеры.

  1. .
  2. .
  3. , так как функции и – бесконечно малые при x→+∞, то , как сумма бесконечно малых функций есть функция бесконечно малая. Функция же является суммой постоянного числа и бесконечно малой функции. Следовательно, по теореме 1 для бесконечно малых функций получаем нужное равенство.

Таким образом, простейшие свойства бесконечно малых и бесконечно больших функций можно записать с помощью следующих условных соотношений: A≠ 0

.

ТЕОРЕМЫ О ПРЕДЕЛАХ

Теорема 1. Предел алгебраической суммы двух, трех и вообще определенного числа функций равен алгебраической сумме пределов этих функций, т.е.

.

Доказательство. Проведем доказательство для двух слагаемых, так как для любого числа слагаемых оно проводится так же. Пусть .Тогда f(x)=b+α(x) и g(x)=c+β(x), где α и β – бесконечно малые функции. Следовательно,

f(x) + g(x)=(b + c) + (α(x) + β(x)).

Так как b + cесть постоянная величина, а α(x) + β(x) – функция бесконечно малая, то

.

Пример. .

Теорема 2. Предел произведения двух, трех и вообще конечного числа функций равен произведению пределов этих функций:

.

Доказательство. Пусть . Следовательно, f(x)=b+α(x) и g(x)=c+β(x) и

fg = (b + α)(c + β) = bc + (bβ + cα + αβ).

Произведение bc есть величина постоянная. Функция bβ + c α + αβ на основании свойств бесконечно малых функций есть величина бесконечно малая. Поэтому .

Следствие 1. Постоянный множитель можно выносить за знак предела:

.

Следствие 2. Предел степени равен степени предела:

.

Пример..

Теорема 3. Предел частного двух функций равен частному пределов этих функций, если предел знаменателя отличен от нуля, т. е.

.

Доказательство. Пусть . Следовательно, f(x)=b+α(x) и g(x)=c+β(x), где α, β – бесконечно малые. Рассмотрим частное

.

Дробь является бесконечно малой функцией, так как числитель есть бесконечно малая функция, а знаменатель имеет предел c2≠0.

Примеры.

  1. .
  2. .
  3. Рассмотрим . При x→1 числитель дроби стремится к 1, а знаменатель стремится к 0. Но так как , т.е. есть бесконечно малая функция при x→1, то .

Теорема 4. Пусть даны три функции f(x), u(x) и v(x), удовлетворяющие неравенствам u(x)≤f(x)≤ v(x). Если функции u(x) и v(x) имеют один и тот же предел при x→a (или x→∞), то и функция f(x) стремится к тому же пределу, т.е. если

, то .

Смысл этой теоремы понятен из рисунка.

Доказательство теоремы 4 можно найти, например, в учебнике: Пискунов Н. С. Дифференциальное и интегральное исчисления, т. 1 – М.: Наука, 1985.

Теорема 5. Если при x→a (или x→∞) функция y=f(x) принимает неотрицательные значения y≥0 и при этом стремится к пределу b, то этот предел не может быть отрицательным: b≥0.

Доказательство. Доказательство проведем методом от противного. Предположим, что b<0, тогда |y – b|≥|b| и, следовательно, модуль разности не стремится к нулю при x→a. Но тогда y не стремится к пределу b при x→a, что противоречит условию теоремы.

Теорема 6. Если две функции f(x) и g(x) при всех значениях аргумента x удовлетворяют неравенству f(x)≥ g(x) и имеют пределы , то имеет место неравенство b≥c.

Доказательство. По условию теоремы f(x)-g(x) ≥0, следовательно, по теореме 5 , или .

ОДНОСТОРОННИЕ ПРЕДЕЛЫ

До сих пор мы рассматривали определение предела функции, когда x→a произвольным образом, т.е. предел функции не зависел от того, как располагалось x по отношению к a, слева или справа от a. Однако, довольно часто можно встретить функции, которые не имеют предела при этом условии, но они имеют предел, если x→a, оставаясь с одной стороны от а, слева или справа (см. рис.). Поэтому вводят понятия односторонних пределов.

Если f(x) стремится к пределу b при x стремящемся к некоторому числу a так, что xпринимает только значения, меньшие a, то пишут и называют bпределом функции f(x) в точке a слева.

Таким образом, число b называется пределом функции y=f(x) при x→aслева, если каково бы ни было положительное число ε, найдется такое число δ (меньшее a), что для всех выполняется неравенство .

Аналогично, если x→a и принимает значения большие a, то пишут и называют b пределом функции в точке а справа. Т.е. число b называется пределом функции y=f(x) при x→a справа, если каково бы ни было положительное число ε, найдется такое число δ (большее а), что для всех выполняется неравенство .

Заметим, что если пределы слева и справа в точке a для функции f(x) не совпадают, то функция не имеет предела (двустороннего) в точке а.

Примеры.

  1. Рассмотрим функцию y=f(x), определенную на отрезке [0,1] следующим образом

    Найдем пределы функции f(x) при x→3. Очевидно, , а .

  2. .
  3. .

ТИПЫ НЕОПРЕДЕЛЕННОСТЕЙ И СПОСОБЫ ИХ РАСКРЫТИЯ

Часто при вычислении пределов какой-либо функции, непосредственное применение теорем о пределах не приводит к желаемой цели. Так, например, нельзя применять теорему о пределе дроби, если ее знаменатель стремится к нулю. Поэтому часто прежде, чем применять эти теоремы, необходимо тождественно преобразовать функцию, предел которой мы ищем.

Условные выражения

характеризуют типы неопределенностей и применяются для обозначения переменных величин, при вычислении предела которых нельзя сразу применять общие свойства пределов.

Рассмотрим некоторые приемы раскрытия неопределенностей.

I. Неопределенность .

  1. .
  2. .

    При разложении числителя на множители воспользовались правилом деления многочлена на многочлен «углом». Так как число x=1 является корнем многочлена x3 – 6x2 + 11x– 6, то при делении получим

  3. .

II. Неопределенность .

  1. .

    При вычислении предела числитель и знаменатель данной дроби разделили на x в старшей степени.

  2. .
  3. .
  4. .

При вычислении предела воспользовались равенством ,если x<0.

Следующие виды неопределенностей с помощью алгебраических преобразований функции, стоящей под знаком предела, сводят к одному из рассмотренных выше случаев или .

III. Неопределенность 0 ·∞.

.

IV. Неопределенность ∞ –∞.

  1. .

Интуитивное введение в пределы – лучше объяснение

Пределы

, основы исчисления кажутся такими искусственными и надменными: «Пусть x приближается к 0, но не достигает его, но мы будем действовать так, как будто оно есть…» Ух.

Вот как я научился ими пользоваться:

  • Что такое лимит? Наше лучшее предсказание точки, которую мы не наблюдали.
  • Как сделать прогноз? Увеличьте соседние точки. Если наш прогноз всегда находится между соседними точками, независимо от того, насколько мы увеличиваем масштаб, это наша оценка.
  • Зачем нужны лимиты? В Math есть сценарии «черной дыры» (деление на ноль, уход в бесконечность), и ограничения дают нам оценку, когда мы не можем вычислить результат напрямую.
  • Как узнать, что мы правы? Нет. Наше предсказание, предел, не обязательно должно соответствовать действительности. Но для большинства природных явлений так оно и есть.

Лимиты зададим вопрос «А что, если?». Если мы можем непосредственно наблюдать функцию при значении (например, x = 0 или x бесконечно растущем), нам не нужен прогноз.Предел задается вопросом: «Если вы можете видеть все , кроме , одно значение, как вы думаете, что там?».

Когда наш прогноз непротиворечив и улучшается, чем ближе мы смотрим на , мы чувствуем в нем уверенность. И если функция работает плавно, как большинство реальных функций, предел – это место, где должна быть отсутствующая точка.

Ключевая аналогия: предсказание футбольного мяча

Представьте, что вы смотрите футбольный матч. К сожалению, соединение нестабильно:

Ack! Мы пропустили то, что произошло в 4:00.Даже в этом случае, как вы прогнозируете положение мяча?

Легко. Просто возьмите соседние моменты (3:59 и 4:01) и предсказывайте, что мяч окажется где-то посередине.

И… работает! Объекты реального мира не телепортируются; они перемещаются через промежуточные позиции на своем пути от A до B. Наш прогноз: «В 4:00 мяч находился между его положением в 3:59 и 4:01». Неплохо.

С камерой замедленной съемки мы могли бы даже сказать: «В 4:00 мяч находился между своими позициями в 3:59.999 и 4: 00.001 ».

Наш прогноз кажется верным. Можем ли мы сформулировать почему?

  • Прогнозы совпадают при увеличении масштабов . Представьте, что диапазон 3: 59–4: 01 составлял 9,9–10,1 метра, но после увеличения до 3: 59,999–4: 00.001 диапазон расширился до 9–12 метров. Ой ой! Масштабирование должно сузить до нашей оценки, а не ухудшить ее! Не каждый уровень масштабирования должен быть точным (представьте, что вы смотрите игру каждые 5 минут), но чтобы чувствовать себя уверенно, должен быть некоторый порог, при котором последующие увеличения только усиливают нашу оценку диапазона.

  • Согласен до и после. Представьте, что в 3:59 мяч летел на 10 метров и катился вправо, а в 4:01 он был на 50 метрах и катился влево. Что случилось? У нас был внезапный прыжок (смена камеры?), И теперь мы не можем определить положение мяча. У кого был мяч в 4:00? Эта двусмысленность лишает нас возможности делать уверенные прогнозы.

С учетом этих требований можно сказать: «В 4:00 мяч был на расстоянии 10 метров. Эта оценка подтверждается нашим первоначальным увеличением (3: 59-4: 01, которое составляет 9.От 9 до 10,1 метра) и следующего (3: 59.999-4: 00.001, что составляет от 9,999 до 10,001 метра) ».

Лимиты – это стратегия для уверенных прогнозов.

Изучение интуиции

Давайте пока не будем приводить математические определения. Для каких вещей в реальном мире мы хотим получить точный прогноз, но не можем легко измерить?

Какова длина окружности?

Найти Пи «экспериментально» сложно: выбить строку и линейку?

Мы не можем измерить форму с кажущимися бесконечными сторонами, но мы можем задаться вопросом: «Есть ли предсказанное значение числа Пи, которое всегда будет точным, если мы продолжаем увеличивать стороны?»

Архимед выяснил, что число пи имеет диапазон

, используя такой процесс:

Это было предшественником исчисления: он определил, что пи – это число, которое остается между его постоянно сужающимися границами. В настоящее время у нас есть современные предельные определения числа пи.

Как выглядит идеально непрерывный рост?

e, одно из моих любимых чисел, можно определить так:

Мы не можем легко измерить результат бесконечно сложного роста. Но если мы могли бы сделать прогноз , есть ли какая-то единственная ставка, которая когда-либо была бы точной? Вроде около 2,71828…

Можем ли мы использовать простые формы для измерения сложных?

Круги и кривые сложно измерить, но прямоугольники – легко.Если бы мы могли использовать бесконечное количество прямоугольников для имитации искривленной области, сможем ли мы получить результат, выдерживающий бесконечную проверку? (Возможно, нам удастся найти площадь круга.)

Можем ли мы узнать скорость в мгновение ока?

Скорость – это забавно: для этого необходимо измерение до и после (пройденное расстояние / затраченное время), но разве мы не можем определить скорость в отдельные моменты времени? Грм.

Пределы помогают решить эту загадку: спрогнозируйте свою скорость, путешествуя в соседний момент.Затем задайте «невозможный вопрос»: какова ваша прогнозируемая скорость, когда разрыв с соседним моментом равен нулю?

Примечание. Предел – не панацея. Мы не можем предположить, что он существует, и не может быть ответа на каждый вопрос. Например: число целых чисел четное или нечетное? Количество бесконечно, и ни «четное», ни «нечетное» предсказание не остается точным, когда мы считаем больше. Никакого надежного прогноза не существует.

Для пи, е и основ исчисления умные умы провели доказательства, чтобы определить: «Да, наши предсказанные значения становятся тем точнее, чем ближе мы смотрим.«Теперь я понимаю, почему так важны: это знак одобрения наших прогнозов.

Математика: формальное определение предела

Пределы – это хорошо подтвержденные прогнозы. Вот официальное определение:

означает, что для всех действительных ε> 0 существует вещественное δ> 0 такое, что для всех x с 0

Давайте сделаем это читаемым:

Математика Английский Человек Английский
Когда мы «строго прогнозируем», что f (c) = L, мы имеем в виду
для всех действительных ε> 0 для любой допустимой погрешности, которую мы хотим (+/-. 1 метр)
существует реальное значение δ> 0 существует уровень масштабирования (+/- 0,1 секунды)
такой, что для всех x с 0, где прогноз остается точным с точностью до погрешность

Здесь есть несколько тонкостей:

  • Уровень масштабирования (дельта, δ) – это вход функции, то есть время в видео
  • Допустимая погрешность (эпсилон, ε) – это максимальное значение, которое выходной сигнал функции (положение мяча) может отличаться от нашего прогноза на всем уровне масштабирования.
  • Условие абсолютного значения (0 <| x - c | <δ) означает, что положительные и отрицательные смещения должны работать, и мы пропускаем саму черную дыру (когда | x - c | = 0).

Мы не можем оценить ввод черной дыры, но можем сказать: «За исключением отсутствующей точки, весь уровень масштабирования подтверждает прогноз $ f (c) = L $». И поскольку $ f (c) = L $ выполняется для , любой допустимый предел ошибки , который мы можем найти, мы чувствуем уверенно.

Можно ли сделать несколько прогнозов? Представьте, что мы предсказали L1 и L2 для f (c). Между ними есть некоторая разница (назовем ее .1), поэтому есть некоторая погрешность (0,01), которая поможет выявить более точную. Выход каждой функции в диапазоне не может быть в пределах.01 обоих прогнозов. У нас либо есть единый бесконечно точный прогноз, либо его нет.

Да, мы можем проявить симпатию и попросить «левый предел» (прогноз до события) и «правый предел» (прогноз после события), но у нас есть реальный предел только тогда, когда они согласны.

Функция является непрерывной, если она всегда соответствует предсказанному значению (и прерывистой, если нет):

Исчисление обычно изучает непрерывные функции, играя в игру «Мы делаем прогнозы, но только потому, что знаем, что они верны.”

Математика: показывает, что предел существует

У нас есть требования для надежного прогноза. Вопросы, просящие вас «доказать, что предел существует», требуют, чтобы вы обосновали свою оценку.

Например: докажите, что предел при x = 2 существует для

Первая проверка: нужен ли вообще лимит? К сожалению, мы это делаем: просто вставка «x = 2» означает, что у нас есть деление на ноль. Drats.

Но интуитивно мы видим, что один и тот же «ноль» (x – 2) может быть отменен сверху и снизу.Вот как танцевать это опасное танго:

  • Предположим, что x равно , кроме 2 (Должно быть! Мы делаем прогноз извне.)
  • Затем мы можем отменить (x – 2) сверху и снизу, так как оно не равно нулю.
  • Осталось f (x) = 2x + 1. Эту функцию можно использовать за пределами черной дыры.
  • Что предсказывает эта более простая функция? Это f (2) = 2 * 2 + 1 = 5.

Итак, f (2) = 5 – это наш прогноз. Но вы заметили коварство? Мы притворились, что x не 2 [чтобы разделить (x-2)], а затем подключили 2 после того, как этот проблемный элемент исчез! Подумайте об этом так: мы использовали простое поведение из вне события , чтобы предсказать грубое поведение в событии .

Мы можем доказать, что эти махинации дают надежный прогноз, и что f (2) = 5 бесконечно точное.

Для любого порога точности (ε) нам нужно найти «диапазон масштабирования» (δ), при котором мы остаемся в пределах заданной точности. Например, можем ли мы сохранить оценку в пределах +/- 1,0?

Конечно. Нам нужно узнать, где

т.

Другими словами, x должен оставаться в пределах 0,5 из 2, чтобы поддерживать начальное требование точности 1,0. Действительно, когда x находится между 1.5 и 2.5, f (x) изменяется от f (1.5) = 4 до f (2.5) = 6, оставаясь +/- 1.0 от нашего предсказанного значения 5.

Мы можем сделать обобщение для любого допуска ошибок (ε), подключив его к 1.0 выше. Получаем:

Если наш уровень масштабирования равен «δ = 0,5 * ε», мы останемся в пределах исходной ошибки. Если наша ошибка равна 1.0, нам нужно увеличить до .5; если 0,1, нам нужно увеличить до 0,05.

Эта простая функция была удобным примером. Идея состоит в том, чтобы начать с начального ограничения (| f (x) – L | <ε), подключить f (x) и L и найти расстояние от точки черной дыры (| x - c | < ?).Часто это упражнение по алгебре.

Иногда вас просят просто найти предел (вставьте 2 и получите f (2) = 5), в других случаях вас просят доказать, что предел существует, то есть прокрутить эпсилон-дельта-алгебру.

Переворачивание нуля и бесконечности

Infinity, когда используется в качестве лимита, означает «растет без остановки». Символ ∞ не более чем число, чем предложение «растет без остановки» или «мой запас трусов истощается». Это понятия, а не числа (для нашего уровня математики, только Алеф я).

При использовании ∞ в ограничении мы спрашиваем: «Если x растет без остановки, можем ли мы сделать прогноз, который останется точным?». Если есть предел, это означает, что прогнозируемое значение всегда подтверждается, как бы далеко мы не смотрели.

Но я все еще не люблю бесконечность, потому что не вижу ее. Но я вижу ноль. С ограничениями можно

переписать

как

Можно хитростью определить y = 1 / x, заменить элементы в формуле, а затем использовать

, так что снова похоже на нормальную проблему! (Примечание Тима в комментариях: предел идет справа, так как x стремится к положительной бесконечности).Я предпочитаю эту схему, потому что я вижу место, к которому мы сужаемся (у нас всегда заканчивается бумага при построении бесконечной версии).

Почему пределы не используются чаще?

Представьте себе ребенка, который понял, что «ставим ноль на конце» делает число в 10 раз больше. Есть 5? Запишите «5», затем «0» или 50. Есть 100? Сделайте 1000. И так далее.

Он не понял, почему работает умножение, почему это правило оправдано … но, согласитесь, он уверен, что умеет умножать на 10.2 $ »без строгого обоснования. Тем не менее, судя по его неофициальным результатам, двигатели вращаются, а самолеты летают.

Ошибка педагогики математического анализа создает препятствие, подобное тому, что «вы должны знать пределы ™, прежде чем научиться ценить математический анализ», хотя очевидно, что изобретатели математического анализа этого не знали. Я бы предпочел эту прогрессию:

  • Исчисление задает, казалось бы, невозможные вопросы: когда прямоугольники могут измерять кривую? Можем ли мы обнаружить мгновенное изменение?
  • Пределы дают стратегию ответа на «невозможные» вопросы («Если вы можете сделать прогноз, выдерживающий бесконечную проверку, мы скажем, что это нормально.2 $), точно так же, как мы запоминаем ярлыки для правил, которые мы проверили с умножением (добавление нуля означает умножение на 10). Но все же приятно знать, почему короткие пути оправданы.

Пределы – не единственный инструмент для проверки ответов на невозможные вопросы; бесконечно малые тоже работают. Ключом является понимание того, что мы пытаемся предсказать, , затем , изучение правил построения прогнозов.

Счастливая математика.

Другие сообщения из этой серии

  1. Нежное введение в изучение исчисления
  2. Понимание исчислений с помощью метафоры банковского счета
  3. Доисторическое исчисление: открытие числа Пи
  4. Аналогия с исчислением: интегралы как умножение
  5. Исчисление: построение интуиции для производных
  6. Как понять деривативы: правила продукта, власти и цепочки
  7. Как понимать производные: правило частного, экспоненты и логарифмы
  8. Интуитивное знакомство с ограничениями
  9. Интуиция к серии Тейлора (аналогия с ДНК)
  10. Зачем нужны пределы и бесконечно малые?
  11. Исчисление обучения: преодоление нашей искусственной потребности в точности
  12. Дружеский чат о том, 0. 999 … = 1
  13. Аналогия: камера исчисления
  14. Практика абстракции: графы исчисления
  15. Quick Insight: более простая арифметика с исчислением
  16. Как сложить от 1 до 100 с помощью исчисления
  17. Интеграл греха (x): геометрическая интуиция

1.1: Введение в понятие лимита

Пределы

Две ключевые проблемы привели к первоначальной формулировке исчисления:

(1) касательная задача, или как определить наклон прямой, касательной к кривой в точке;

и (2) проблема площади, или как определить площадь под кривой.

Концепция предельного или ограничивающего процесса, необходимая для понимания исчисления, существует уже тысячи лет. Фактически, ранние математики использовали процесс ограничения для получения более точных приближений площадей кругов. Тем не менее, формальное определение предела – в том виде, в каком мы его знаем и понимаем сегодня – появилось только в конце 19 века. Поэтому мы начинаем наши поиски понимания пределов, как это делали наши математические предки, с интуитивного подхода. 2} \),

, которые показаны на рисунке \ (\ PageIndex {1} \).В частности, давайте сосредоточим наше внимание на поведении каждого графа в точке \ (x = 2 \) и вокруг нее.

Рисунок \ (\ PageIndex {1} \): Эти графики показывают поведение трех различных функций вокруг \ (x = 2 \).

Каждая из трех функций не определена в \ (x = 2 \), но если мы сделаем это утверждение, а не другое, мы дадим очень неполную картину того, как каждая функция ведет себя в окрестности \ (x = 2 \). Чтобы более полно выразить поведение каждого графа в окрестности \ (2 \), нам нужно ввести понятие предела.2−4) / (x − 2) \) ведет себя около \ (x = 2 \) на рисунке \ (\ PageIndex {1} \). Поскольку значения \ (x \) приближаются к \ (2 \) с любой стороны от \ (2 \), значения \ (y = f (x) \) приближаются к \ (4 \). Математически мы говорим, что предел \ (f (x) \) при приближении \ (x \) к \ (2 \) равен \ (4 \). Символически мы выражаем этот предел как

.

\ (\ Displaystyle \ lim_ {х \ к 2} е (х) = 4 \).

Из этого очень краткого неформального взгляда на один предел, давайте начнем с разработки интуитивного определения предела . Мы можем думать о пределе функции при числе a как о единственном действительном числе \ (L \), к которому функциональные значения приближаются, как \ (x \) – значения приближаются к a, при условии, что такое действительное число \ (L \ ) существуют.Если говорить более внимательно, мы имеем следующее определение:

Определение (интуитивно понятное): предел

Пусть \ (f (x) \) будет функцией, определенной для всех значений в открытом интервале, содержащем \ (a \), за возможным исключением самого a, и пусть \ (L \) будет действительным числом. Если все значения функции \ (f (x) \) приближаются к действительному числу \ (L \), а значения \ (x (≠ a) \) приближаются к числу a, то мы говорим, что предел \ ( f (x) \) по мере приближения \ (x \) к \ (a \) равно \ (L \). (Более сжато, поскольку \ (x \) приближается к \ (a \), \ (f (x) \) приближается и остается близко к \ (L \).) Условно мы выражаем эту идею как

\ [\ lim_ {x \ to a} f (x) = L. \]

Мы можем оценить пределы, составив таблицы функциональных значений и посмотрев на их графики. Этот процесс описан в следующей стратегии решения проблем.

Стратегия решения проблем: оценка предела с помощью таблицы функциональных значений

1. Чтобы оценить \ (\ displaystyle \ lim_ {x \ to a} f (x) \), мы начинаем с заполнения таблицы функциональных значений. Мы должны выбрать два набора значений \ (x \) – один набор значений, приближающихся к \ (a \) и меньших, чем \ (a \), и другой набор значений, приближающихся к \ (a \) и больших, чем \ (a \).Таблица \ (\ PageIndex {1} \) демонстрирует, как могут выглядеть ваши таблицы.

Таблица \ (\ PageIndex {1} \)
\ (х \) \ (f (x) \) \ (х \) \ (f (x) \)
\ (а-0,1 \) \ (f (a-0.1) \) \ (а + 0,1 \) \ (f (a + 0,1) \)
\ (а-0.01 \) \ (f (а-0,01) \) \ (а + 0,001 \) \ (f (a + 0,001) \)
\ (а-0,001 \) \ (f (a-0,001) \) \ (а + 0,0001 \) \ (f (a + 0,001) \)
\ (а-0,0001 \) \ (ф (а-0,0001) \) \ (а + 0,00001 \) \ (f (а + 0,0001) \)
При необходимости используйте дополнительные значения. При необходимости используйте дополнительные значения.

2. Затем давайте посмотрим на значения в каждом из столбцов \ (f (x) \) и определим, приближаются ли значения к одному значению при движении вниз по каждому столбцу. В наших столбцах мы смотрим на последовательность \ (f (a − 0.1) \), \ (f (a − 0.01) \), \ (f (a − 0.001) \), \ (f (a − 0.0001) \) и т. д., а также \ (f (a + 0,1), \; f (a + 0,01), \; f (a + 0,001), \; f (a + 0,0001) \) и т. д. (Примечание: хотя мы выбрали \ (x \) – значения \ (a ± 0.1, \; a ± 0,01, \; a ± 0,001, \; a ± 0,0001 \) и т. Д., И эти значения, вероятно, будут работать почти каждый раз, в очень редких случаях нам может потребоваться изменить наш выбор.)

3. Если оба столбца приближаются к общему \ (y \) – значению \ (L \), мы утверждаем \ (\ displaystyle \ lim_ {x \ to a} f (x) = L \). Мы можем использовать следующую стратегию для подтверждения результата, полученного из таблицы, или в качестве альтернативного метода оценки лимита.

4. Используя графический калькулятор или компьютерное программное обеспечение, которое позволяет нам строить графики функций, мы можем построить график функции \ (f (x) \), убедившись, что функциональные значения \ (f (x) \) для \ (x \) -значения возле a находятся в нашем окне.Мы можем использовать функцию трассировки, чтобы перемещаться по графику функции и наблюдать за считыванием значения \ (y \) по мере приближения значений \ (x \) к a. Если \ (y \) – значения приближаются к \ (L \), как наши \ (x \) – значения приближаются к \ (a \) с обоих направлений, то \ (\ displaystyle \ lim_ {x \ to a} f (x ) = L \). Возможно, нам придется увеличить масштаб нашего графика и повторить этот процесс несколько раз.

Мы применяем эту стратегию решения проблем для вычисления предела в примерах \ (\ PageIndex {1A} \) и \ (\ PageIndex {1B} \).

Пример \ (\ PageIndex {1A} \): оценка предела с помощью таблицы функциональных значений

Оцените \ (\ displaystyle \ lim_ {x \ to 0} \ frac {\ sin x} {x} \), используя таблицу функциональных значений.

Решение

Мы вычислили значения \ (f (x) = \ dfrac {\ sin x} {x} \) для значений \ (x \), перечисленных в таблице \ (\ PageIndex {2} \).

Таблица \ (\ PageIndex {2} \)
\ (х \) \ (\ frac {\ sin x} {x} \) \ (х \) \ (\ frac {\ sin x} {x} \)
-0,1 0.998334166468 0,1 0,998334166468
-0,01 0,999983333417 0,01 0,999983333417
-0,001 0,999999833333 0,001 0,999999833333
-0,0001 0,999999998333 0.0001 0,999999998333

Примечание. Значения в этой таблице были получены с использованием калькулятора и всех позиций, указанных в выходных данных калькулятора.

Читая каждый столбец \ (\ dfrac {\ sin x} {x} \), мы видим, что значения в каждом столбце приближаются к единице. Таким образом, вполне разумно заключить, что \ (\ displaystyle \ lim_ {x \ to0} \ frac {\ sin x} {x} = 1 \). Сгенерированный калькулятором или компьютером график \ (f (x) = \ dfrac {\ sin x} {x} \) будет похож на тот, что показан на рисунке \ (\ PageIndex {2} \), и он подтверждает наши оценивать.

Рисунок \ (\ PageIndex {2} \): график \ (f (x) = (\ sin x) / x \) подтверждает оценку из таблицы.

Пример \ (\ PageIndex {1B} \): оценка предела с помощью таблицы функциональных значений

Оцените \ (\ displaystyle \ lim_ {x \ to4} \ frac {\ sqrt {x} −2} {x − 4} \), используя таблицу функциональных значений.

Решение

Как и раньше, мы используем таблицу – в данном случае Table \ (\ PageIndex {3} \) – для перечисления значений функции для заданных значений \ (x \).

Таблица \ (\ PageIndex {3} \)
\ (х \) \ (\ frac {\ sqrt {x} −2} {x − 4} \) \ (х \) \ (\ frac {\ sqrt {x} −2} {x − 4} \)
3.9 0,251582341869 4,1 0,248456731317
3,99 0,25015644562 4,01 0,24984394501
3,999 0,250015627 4,001 0,249984377
3,9999 0,250001563 4.0001 0,249998438
3.99999 0,25000016 4.00001 0,24999984

Изучив эту таблицу, мы видим, что функциональные значения меньше 4, по-видимому, уменьшаются до 0,25, тогда как функциональные значения больше 4, по-видимому, увеличиваются до 0,25. Мы заключаем, что \ (\ displaystyle \ lim_ {x \ to4} \ frac {\ sqrt {x} −2} {x − 4} = 0.25 \). Мы подтверждаем эту оценку, используя график \ (f (x) = \ dfrac {\ sqrt {x} −2} {x − 4} \), показанный на рисунке \ (\ PageIndex {3} \).

Рисунок \ (\ PageIndex {3} \): график \ (\ frac {\ sqrt {x} −2} {x − 4} \) подтверждает оценку из таблицы

Упражнение \ (\ PageIndex {1} \)

Оцените \ (\ displaystyle \ lim_ {x \ to 1} \ frac {\ frac {1} {x} −1} {x − 1} \) с использованием таблицы функциональных значений. Используйте график, чтобы подтвердить свою оценку.

Подсказка

Используйте 0,9, 0,99, 0.999, 0.9999, 0.99999 и 1.1, 1.01, 1.001, 1.0001, 1.00001 в качестве значений таблицы.

Ответ

\ [\ lim_ {x \ to1} \ frac {\ frac {1} {x} −1} {x − 1} = – 1 \ nonumber \]

На этом этапе мы видим из примеров \ (\ PageIndex {1A} \) и \ (\ PageIndex {1b} \), что может быть так же легко, если не проще, оценить предел функции, проверив ее график, чтобы оценить предел с помощью таблицы функциональных значений. В примере \ (\ PageIndex {2} \) мы оцениваем ограничение исключительно путем просмотра графика, а не использования таблицы функциональных значений.

Пример \ (\ PageIndex {2} \): оценка предела с помощью графика

Для \ (g (x) \), показанного на рисунке \ (\ PageIndex {4} \), оцените \ (\ displaystyle \ lim_ {x \ to − 1} g (x) \).

Рисунок \ (\ PageIndex {4} \): график \ (g (x) \) включает одно значение не на гладкой кривой.

Решение :

Несмотря на то, что \ (g (−1) = 4 \), поскольку \ (x \) – значения приближаются к \ (- 1 \) с любой стороны, значения \ (g (x) \) приближаются к \ ( 3 \). Следовательно, \ (\ displaystyle \ lim_ {x \ to − 1} g (x) = 3 \).Обратите внимание, что мы можем определить этот предел, даже не зная алгебраического выражения функции.

На основе примера \ (\ PageIndex {2} \) мы делаем следующее наблюдение: предел функции может существовать в точке, и функция может быть определена в этой точке, но предел функция и значение функции в точке могут отличаться.

Упражнение \ (\ PageIndex {2} \)

Используйте график \ (h (x) \) на рисунке \ (\ PageIndex {5} \) для вычисления \ (\ displaystyle \ lim_ {x \ to 2} h (x) \), если это возможно.

Рисунок \ (\ PageIndex {5} \):
Подсказка

Какое \ (y \) – значение приближается к функции по мере приближения \ (x \) – значений к \ (2 \)?

Решение

\ (\ Displaystyle \ lim_ {х \ к 2} час (х) = – 1 \)

Просмотр таблицы функциональных значений или графика функции дает нам полезное представление о значении предела функции в данной точке. Однако эти методы слишком полагаются на догадки.В конечном итоге нам необходимо разработать альтернативные методы оценки пределов. Эти новые методы носят более алгебраический характер, и мы исследуем их в следующем разделе; однако на этом этапе мы вводим два особых ограничения, которые лежат в основе будущих техник.

Два важных ограничения

Пусть \ (a \) будет действительным числом, а \ (c \) будет константой.

  1. \ (\ Displaystyle \ lim_ {х \ к а} х = а \)
  2. \ (\ Displaystyle \ lim_ {х \ к а} с = с \)

Мы можем сделать следующие наблюдения об этих двух пределах.

  1. Что касается первого предела, обратите внимание, что по мере того, как \ (x \) приближается к \ (a \), то же самое происходит с \ (f (x) \), потому что \ (f (x) = x \). Следовательно, \ (\ displaystyle \ lim_ {x \ to a} x = a \).
  2. В качестве второго предела рассмотрим таблицу \ (\ PageIndex {4} \).
Таблица \ (\ PageIndex {4} \)
\ (х \) \ (f (x) = c \) \ (х \) \ (f (x) = c \)
\ (а-0.1 \) \ (с \) \ (а + 0,1 \) \ (с \)
\ (а-0,01 \) \ (с \) \ (а + 0,01 \) \ (с \)
\ (а-0,001 \) \ (с \) \ (а + 0,001 \) \ (с \)
\ (а-0,0001 \) \ (с \) \ (а + 0.0001 \) \ (с \)

Обратите внимание, что для всех значений \ (x \) (независимо от того, приближаются ли они к \ (a \)), значения \ (f (x) \) остаются постоянными в \ (c \). У нас нет другого выбора, кроме как заключить \ (\ displaystyle \ lim_ {x \ to a} c = c \).

Существование предела

Когда мы рассматриваем предел в следующем примере, имейте в виду, что для того, чтобы предел функции существовал в точке, функциональные значения должны приближаться к единственному значению действительного числа в этой точке.Если функциональные значения не приближаются к единственному значению, то предел не существует.

Пример \ (\ PageIndex {3} \): оценка несуществующего предела

Оцените \ (\ displaystyle \ lim_ {x \ to 0} \ sin (1 / x) \), используя таблицу значений.

Решение

Таблица \ (\ PageIndex {5} \) перечисляет значения функции \ (\ sin (1 / x) \) для заданных значений \ (x \).

Таблица \ (\ PageIndex {5} \)
\ (х \) \ (\ sin (1 / x) \) \ (х \) \ (\ sin (1 / x) \)
-0.1 0,544021110889 0,1 -0,544021110889
-0,01 0,50636564111 0,01 -0,50636564111
-0,001 -0,8268795405312 0,001 0,8268795405312
-0,0001 0.305614388888 0,0001 -0,305614388888
-0,00001 -0,035748797987 0,00001 0,035748797987
-0,000001 0,349993504187 0,000001 -0,349993504187

Изучив таблицу функциональных значений, мы можем увидеть, что значения \ (y \), похоже, не приближаются к какому-либо одному значению.Похоже, предела не существует. Прежде чем сделать такой вывод, давайте рассмотрим более систематический подход. Возьмем следующую последовательность \ (x \) – значений, приближающихся к \ (0 \):

\ [\ frac {2} {π}, \; \ frac {2} {3π}, \; \ frac {2} {5π}, \; \ frac {2} {7π}, \; \ frac { 2} {9π}, \; \ frac {2} {11π}, \;…. \ Nonumber \]

Соответствующие \ (y \) – значения равны

\ [1, \; – 1, \; 1, \; – 1, \; 1, \; – 1, \; …. \ nonumber \]

На этом этапе мы действительно можем заключить, что \ (\ displaystyle \ lim_ {x \ to 0} \ sin (1 / x) \) не существует.(Математики часто сокращают «не существует» как DNE. Таким образом, мы бы написали \ (\ displaystyle \ lim_ {x \ to 0} \ sin (1 / x) \) DNE.) График \ (f (x) = \ sin (1 / x) \) показано на рисунке \ (\ PageIndex {6} \) и дает более ясную картину поведения \ (\ sin (1 / x) \) как \ (x \) приближается к \ (0 \). Вы можете видеть, что \ (\ sin (1 / x) \) все более резко колеблется между \ (- 1 \) и \ (1 \), когда \ (x \) приближается к \ (0 \).

Рисунок \ (\ PageIndex {6} \): график \ (f (x) = \ sin (1 / x) \) быстро колеблется между \ (- 1 \) и \ (1 \) как \ (x \ ) приближается к \ (0 \).2−4∣} {x − 2} \) не существует.

index-of.es/

 Название Размер
 Android / -
 Галерея искусств/                  -
 Атаки / -
 Переполнение буфера / -
 C ++ / -
 CSS / -
 Компьютер / -
 Конференции / -
 Растрескивание / -
 Криптография / -
 Базы данных / -
 Глубокая сеть / -
 Отказ в обслуживании/            -
 Электронные книги / -
 Перечисление / -
 Эксплойт / -
 Техники неудачной атаки / -
 Судебно-медицинская экспертиза / -
 Галерея / -
 HTML / -
 Взломать / -
 Взлом-веб-сервер / -
 Взлом беспроводных сетей / -
 Взлом / -
 Генератор хешей / -
 JS / -
 Ява/                         -
 Linux / -
 Отмыкание/                  -
 Журналы / -
 Вредоносное ПО / -
 Метасплоит / -
 Разное / -
 Разное / -
 Протоколы сетевой безопасности / -
 Сеть / -
 ОПЕРАЦИОННЫЕ СИСТЕМЫ/                           -
 Другое / -
 PHP / -
 Perl / -
 Программирование / -
 Python / -
 RSS / -
 Rdbms / -
 Разобрать механизм с целью понять, как это работает/          -
 Рубин/                         -
 Сканирование сетей / -
 Безопасность/                     -
 Захват сеанса / -
 Снифферы / -
 Социальная инженерия/           -
 Поддерживает / -
 Системный взлом / -
 Инструменты/                        -
 Учебники / -
 UTF8 / -
 Unix / -
 Вариос-2 / -
 Варианты / -
 Видео/                       -
 Вирусы / -
 Окна / -
 Беспроводная связь / -
 Xml / -
 z0ro-Репозиторий-2 / -
 z0ro-Репозиторий-3 / -
 

Business Calculus

Примечание: видео для разделов 2.1-2,5 были записаны на основе более старого издания книги. Это означает, что некоторые из упомянутых мной номеров разделов больше не будут соответствовать одному и тому же материалу, и скриншоты могут выглядеть по-разному. Однако контент по сути тот же, и я попытался разместить видео в правильном месте в зависимости от того, куда был перемещен материал.

Введение

Идея Precalculus: наклон и скорость изменения

Наклон линии измеряет, насколько быстро линия поднимается или опускается, когда мы движемся по ней слева направо.Он измеряет скорость изменения координаты y по отношению к изменениям координаты x. Если линия представляет собой, например, расстояние, пройденное с течением времени, то ее наклон представляет собой скорость. На рисунке вы можете напомнить себе, как мы рассчитываем уклон, используя две точки на линии:

\ (m = \ text {Наклон от \ (P \) до \ (Q \)} = \ dfrac {\ text {rise}} {\ text {run}} = \ dfrac {y_2-y_1} {x_2-x_1 } = \ dfrac {\ Delta y} {\ Delta x} \)

Мы хотели бы иметь возможность получать такую ​​же информацию (насколько быстро кривая поднимается или опускается, скорость на расстоянии), даже если график не является прямая линия.Но что произойдет, если мы попытаемся найти наклон кривой, как показано на рисунке ниже?

Нам нужны две точки, чтобы определить наклон прямой. Как мы можем найти наклон кривой только в одной точке? Ответ, как показано на рисунке, состоит в том, чтобы найти наклон касательной к кривой в этой точке. У большинства из нас есть интуитивное представление о том, что такое касательная линия. К сожалению, «касательную» сложно определить точно.

Определение (Секущая линия)

Секущая – линия между двумя точками на кривой

Смотрите изображение ниже:

Определение “Не совсем-еще” (касательная линия)

Касательная линия – это линия в одной точке кривой… которая лучше всего может быть кривой в этой точке?

Как вы можете видеть на изображении ниже, чем ближе точка \ (Q \) к точке \ (P \), тем ближе секущий уклон приближается к касательному.Это будет ключом к нахождению касательного наклона, но сначала нам нужно более тщательно определить идею приближения к .

Пределы

В последнем разделе мы увидели, что по мере того, как интервал, на котором мы рассчитывали, уменьшался, секущие склоны приближались к касательной. Предел дает нам лучший язык для обсуждения идеи «подходов».

Предел функции описывает поведение функции, когда переменная близка к заданному номеру, но не равна ему (см. Рисунок ниже).

Определение (предел)

Если значения \ (f (x) \) становятся все ближе и ближе, настолько близко, насколько мы хотим, к одному числу \ (L \), поскольку мы берем значения \ (x \), очень близкие к (но не равные ) число \ (c \), тогда мы говорим: « предел \ (f (x) \), когда \ (x \) приближается к \ (c \) равен \ (L \) », и мы пишем \ [ \ lim \ limits_ {x \ to c} f (x) = L. \] Символ «\ (\ to \)» означает «приближается» или, менее формально, «становится очень близко к».

(Это определение лимита не сформулировано так формально, как могло бы быть, но его достаточно для наших целей в этом курсе.)

Примечание:

  • \ (f (c) \) – единственное число, которое описывает поведение (значение) \ (f (x) \) в точке \ (x = c \).
  • \ (\ lim \ limits_ {x \ to c} f (x) \) – это единственное число, которое описывает поведение \ (f (x) \) рядом, , но НЕ в , точке \ (x = с \).

Если у нас есть график функции вблизи x = c, то обычно легко определить \ (\ lim \ limits_ {x \ to c} f (x) \).

(Вот ссылка на изображения, используемые в следующем видео, а также где-либо еще в этой главе: Графики для пределов и примеры непрерывности.)

Для просмотра этого видео включите JavaScript и рассмотрите возможность обновления до веб-браузера, который поддерживает видео HTML5

Пример 1

Используйте график \ (y = f (x) \) на рисунке ниже, чтобы определить следующие пределы:

  1. \ (\ lim \ limits_ {x \ to 1} f (x) \)
  2. \ (\ lim \ limits_ {x \ to 2} f (x) \)
  3. \ (\ lim \ limits_ {x \ to 3} f (x) \)
  4. \ (\ lim \ limits_ {x \ to 4} f (x) \)
  1. Когда \ (x \) очень близко к 1, значения \ (f (x) \) очень близки к \ (y = 2 \).В этом примере случается, что \ (f (1) = 2 \), но это нерелевантно для предела. Единственное, что имеет значение, – это то, что происходит для \ (x \) , близкого к 1, но \ (x \ neq 1 \).
  2. \ (f (2) \) не определено, но нас интересует только поведение \ (f (x) \) для \ (x \) , близкого к 2, но не равно 2. Когда \ ( x \) близко к 2, значения \ (f (x) \) близки к 3. Если мы ограничим \ (x \) достаточно близко к 2, значения \ (y \) будут как можно ближе к 3, как мы хотим, поэтому \ (\ lim \ limits_ {x \ to 2} f (x) = 3 \).
  3. Когда \ (x \) близко к 3 (или «когда \ (x \) приближается к значению 3»), значения \ (f (x) \) близки к 1 (или «приближаются к значению 1» ), поэтому \ (\ lim \ limits_ {x \ to 3} f (x) = 1 \). Для этого предела совершенно несущественно, что \ (f (3) = 2 \). Нас интересует только то, что происходит с \ (f (x) \) для \ (x \), близкого к 3, но не равного ему.
  4. Это сложнее, и нам нужно быть осторожными. Когда \ (x \) близко к 4 и немного меньше 4 (\ (x \) находится чуть левее 4 на оси \ (x \)), тогда значения \ (f (x) \ ) близки к 2.Но если \ (x \) близко к 4 и немного больше 4, тогда значения \ (f (x) \) близки к 3. Если мы только знаем, что \ (x \) очень близко к 4, тогда мы не можем сказать, будет ли \ (y = f (x) \) близко к 2 или близко к 3 – это зависит от того, находится ли \ (x \) справа или слева от 4. В этой ситуации \ (f (x) \) значения не близки к одному числу, поэтому мы говорим, что \ (f (x) \) не существует. Неважно, что \ (f (4) = 1 \). Предел, поскольку \ (x \) приближается к 4, все равно будет неопределенным, если \ (f (4) \) было 3 или 2 или что-то еще.2-х-1} {х-1} \).

    Вы можете попытаться оценить в \ (x = 1 \), но \ (f (x) \) не определен в \ (x = 1 \). Заманчиво, , но неверно , сделать вывод, что эта функция не имеет предела, поскольку \ (x \) приближается к 1.

    Используя таблицы: Пробуя некоторые “тестовые” значения для x, которые становятся все ближе и ближе к 1 как слева, так и справа, мы получаем

    \ (х \) \ (е (х) \)
    0,9 2. 2 -х-1} {х-1} = 3.2-x-1} {x-1} \) для \ (x \), близкого к 1:

    Обратите внимание, что всякий раз, когда \ (x \) близко к 1, значения \ (y = f (x) \) близки к 3. Поскольку \ (f \) не определено в \ (x = 1 \), в графике есть дыра над \ (x = 1 \), но нас волнует только то, что \ (f (x) \) делает для \ (x \), близкого к , но не равного 1.

    Для просмотра этого видео включите JavaScript и рассмотрите возможность обновления до веб-браузера, который поддерживает видео HTML5

    Односторонние ограничения

    Иногда то, что происходит с нами в каком-либо месте, зависит от того, в каком направлении мы приближаемся к этому месту.Если мы подойдем к Ниагарскому водопаду со стороны выше по течению, то мы будем на 182 фута выше и будем беспокоиться иначе, чем если мы подойдем со стороны вниз по течению. Точно так же значения функции около точки могут зависеть от направления, которое мы используем для приближения к этой точке.

    Определение (левый и правый пределы)

    Левый предел \ (f (x) \) при приближении \ (x \) к \ (c \) равен \ (L \), если значения \ (f (x) \) становятся как можно ближе к \ (L \), как мы хотим, когда \ (x \) очень близко и слева от \ (c \) (т.+} f (x) = \ lim \ limits_ {x \ to 1} f (x) = 1. \]

    Для просмотра этого видео включите JavaScript и рассмотрите возможность обновления до веб-браузера, который поддерживает видео HTML5

    Непрерывность

    «Дружественная» функция, в которой нет перерывов или скачков, называется непрерывной. Более формально

    Определение (непрерывность в точке)

    Функция \ (f (x) \) непрерывна в \ (x = a \) тогда и только тогда, когда \ (\ lim \ limits_ {x \ to a} f (x) = f (a) \).

    На приведенном ниже графике показаны некоторые из различных способов поведения функции в точке и рядом с ней, а таблица содержит некоторую числовую информацию о функции и ее поведении.

    \ (а \) \ (ф (а) \) \ (\ lim \ limits_ {x \ to a} f (x) \)
    1 2 2
    2 1 2
    3 2 Не существует (DNE)
    4 Не определено 2

    Основываясь на информации в таблице, мы можем заключить, что \ (f \) непрерывно в 1, поскольку \ (\ lim \ limits_ {x \ to 1} f (x) = 2 = f (1) \) .Мы также можем сделать вывод из информации в таблице, что \ (f \) не является непрерывным в точках 2, 3 или 4, потому что \ (\ lim \ limits_ {x \ to 2} f (x) \ neq f (2) \ ), \ (\ lim \ limits_ {x \ to 3} f (x) \ neq f (3) \) и \ (\ lim \ limits_ {x \ to 4} f (x) \ neq f (4) \).

    Поведение в точках \ (x = 2 \) и \ (x = 4 \) демонстрирует на графике отверстие , иногда называемое устранимым разрывом , поскольку график можно сделать непрерывным, изменив значение одного точка. Поведение в точке \ (x = 3 \) называется разрывом скачка , так как график перескакивает между двумя значениями.Скачкообразные разрывы и вертикальные асимптоты являются неустранимыми разрывами , потому что они не могут быть исправлены путем изменения значения одной точки.

    Итак, какие функции являются непрерывными? Оказывается, практически каждая функция, которую вы изучали, является непрерывной там, где она определена: полиномиальные, радикальные, рациональные, экспоненциальные и логарифмические функции непрерывны там, где они определены . 3- 4 (2) = 0.\]

  5. Данная функция является рациональной. Он не определен при x = -3, но мы берем предел, когда x приближается к 2, и функция определена в этой точке, поэтому мы можем использовать прямую замену: \ [\ lim \ limits_ {x \ to 2} \ dfrac {x-4} {x + 3} = \ dfrac {2-4} {2 + 3} = – \ dfrac {2} {5}. \]
  6. Эта функция не определена при x = 2 и поэтому не является непрерывной при x = 2. Мы не можем использовать прямую подстановку.
  7. Предварительное вычисление

    : как рассчитать пределы для различных функций

    Аналогично, если x приближается к -∞, функция f в этом случае также становится произвольно близкой к нулю.

    Но нам не нужно ограничивать пределы крайними значениями независимой переменной. А как насчет других интересных особенностей этой функции? Давайте рассмотрим, как работать с ограничениями, когда мы приближаемся к x = 0. Во-первых, обратите внимание, что предел зависит от направления, с которого приближается значение x .

    В этом случае, если мы начнем с левой стороны и приблизимся к x = 0, функция f станет все меньше и меньше (в смысле большей величины, но с отрицательным знаком) без ограничений. Чтобы представить эту ситуацию символически, мы называем предел -∞, но это просто означает, что функция не имеет минимального действительного значения. Мы используем небольшой верхний индекс “-“, чтобы указать подход слева:

    Когда мы приближаемся к x = 0 справа (обозначено верхним индексом «+»), f становится произвольно большим, поэтому мы говорим, что предел равен ∞.

    Мы также можем посмотреть предел любой произвольной точки, а не только «особых» точек. Рассмотрим x = 1.

    Итак, давайте еще раз посмотрим на общее выражение для ограничения данной функции f ( x ), поскольку x приближается к некоторой константе c.

    Рассматривая все приведенные выше примеры, мы можем теперь сказать, что если функция f сколь угодно близка (но не обязательно достигает) некоторого значения L , поскольку x приближается к c с любой стороны, тогда L будет предел этой функции для x приближается к c. В этом случае мы говорим, что предел существует. Односторонние ограничения – это когда функция приближается к определенному значению только с одной стороны (как в функции ниже). В этом случае предела не существует; однако мы можем найти односторонние ограничения, как мы это делали выше.

    Часто мы можем определить предел, просто вычислив f ( c ), но очевидно, что это не всегда работает (особенно если c не находится в области f ).Кроме того, нам может потребоваться рассмотреть направление, с которого мы приближаемся к c (для односторонних ограничений). Однако обычно вы можете легко вычислить предел, если вы также посмотрите на график или таблицу значений около x = c.

    Практическая задача: Рассчитайте следующие пределы.

    а. г. г. г.

    Решение: Это помогает сначала построить график функции, чтобы увидеть эти пределы.

    а. Для этого предела рассмотрим значение ln x , поскольку x становится все ближе и ближе к 0. Функция приближается к -∞, поэтому предел равен

    .

    г.В этом случае мы можем просто подключить c к функции. Обратите внимание, что предел равен 0 независимо от направления подхода.

    г. Здесь, поскольку x становится произвольно большим, также становится ln x (т.е. функция не имеет реального максимального значения). Таким образом,

    г.Опять же, в этом случае направление подхода не имеет значения. Мы можем просто подключить и к функции.

    Правила лимитов

    Некоторые ограничения могут включать сложные выражения.В этих и других случаях часто бывает полезно использовать правила, упрощающие вычисления. Ниже приведены основные свойства пределов для произвольных функций f ( x ) и g ( x ) и произвольной постоянной k.

    Эти правила для пределов позволяют нам разбивать сложные выражения на более простые с целью нахождения предела.

    Практическая задача: Опишите словами, почему для многочлена p ( x ) всегда верно следующее.

    Решение: Напомним, что многочлен p ( x ) имеет следующий вид, где значения c i (где i = 0, 1, 2, 3 ,., n ) – константы:

    Поскольку x приближается к k, многочлен (независимо от его формы) приближается к p ( k ), потому что область полинома – это все действительные числа. Таким образом, когда мы имеем дело с пределами для многочленов, мы можем просто подставить предельное значение для x непосредственно в функцию.Математическое доказательство этого факта не слишком сложно, но результат довольно интуитивно понятен.

    Практическая задача: Рассчитайте следующие пределы.

    а. г. г.

    Решение: В каждом случае можно использовать несколько подходов.Один из них – построить график функции и посмотреть на поведение функции вблизи предельного значения x (это часто бывает полезно независимо от того, какой подход вы используете). В качестве альтернативы вы можете использовать правила ограничений и, при необходимости, просто заменить их.

    а. Здесь замена возможна без проблем. Вы также можете использовать правила лимитов.

    г.В данном случае функция является многочленом степени 2. Мы можем просто подставить.

    г. Для этой функции вы не можете напрямую применять правила ограничений и подстановки. Обратите особое внимание на примечание к графику, что значение x приближается справа.

    Применение ограничений: непрерывность

    Давайте рассмотрим более простое применение ограничений: непрерывность функции.Посмотрите еще раз на функцию .

    Интуитивно мы можем сказать, что функция непрерывна (т. Е. В ней нет “разрывов”) слева от x = 0 и справа от x = 0, но не на x = 0. А как насчет более математического определения? Мы можем использовать ограничения. Функция непрерывна в определенной точке x = c , если выполняются все следующие условия:

  8. c находится в домене f

  9. существует

  10. Обратите внимание, что функция является непрерывной на открытом интервале ( a, b ), если она непрерывна во всех точках этого интервала.

    Практическая задача: Определите, является ли функция непрерывной в данной точке.

    а. при x = 1 б. при x = 0 c. ln x при x = e

    Решение: Для проблемы a обратите внимание, что функция равна строке x + 2, за исключением того, что в ней отсутствует точка (1, 3).Интересно, что лимит здесь есть:

    Однако функция не определена при x = 1. Следовательно, мы не можем использовать подстановку, чтобы найти предел. Поскольку функция нарушает одно (фактически нарушает два) условия непрерывности, она не является непрерывной при x = 1.

    Для части b обратите внимание, что ни одно из условий непрерывности не выполняется.Предел не существует (это ∞), и функция не определена при x = 0. Следовательно, функция не является непрерывной при x = 0.

    Для части c обратите внимание, что функция определена как x = e:

    ln e = 1

    Кроме того, предел определен и может быть вычислен путем замены:

    Таким образом, ln x является непрерывным при x = e.

    Поиск пределов: свойства пределов

    Нахождение предела суммы, разницы и произведения

    Построение графика функции или изучение таблицы значений для определения предела может быть обременительным и трудоемким. По возможности более эффективно использовать свойства пределов , которые представляют собой набор теорем для нахождения пределов.

    Знание свойств пределов позволяет нам вычислять пределы напрямую.Мы можем складывать, вычитать, умножать и делить пределы функций, как если бы мы выполняли операции над самими функциями, чтобы найти предел результата. Точно так же мы можем найти предел функции, возведенной в степень, подняв предел до этой степени. Мы также можем найти предел корня функции, взяв корень предела. Используя эти операции над пределами, мы можем найти пределы более сложных функций, найдя пределы их более простых компонентных функций.

    Общее примечание: свойства пределов

    Пусть [latex] a, k, A [/ latex] и [latex] B [/ latex] представляют действительные числа, а [latex] f [/ latex] и [latex] g [/ latex] являются функциями, например что [латекс] \ underset {x \ to a} {\ mathrm {lim}} f \ left (x \ right) = A [/ latex] и [latex] \ underset {x \ to a} {\ mathrm {lim }} g \ left (x \ right) = B [/ латекс].Для существующих и конечных пределов свойства пределов приведены в таблице ниже.

    Константа, к [латекс] \ underset {x \ to a} {\ mathrm {lim}} k = k [/ latex]
    Постоянное умножение на функцию [латекс] \ underset {x \ to a} {\ mathrm {lim}} \ left [k \ cdot f \ left (x \ right) \ right] = k \ underset {x \ to a} {\ mathrm { lim}} f \ left (x \ right) = kA [/ латекс]
    Сумма функций [латекс] \ underset {x \ to a} {\ mathrm {lim}} \ left [f \ left (x \ right) + g \ left (x \ right) \ right] = \ underset {x \ to a } {\ mathrm {lim}} f \ left (x \ right) + \ underset {x \ to a} {\ mathrm {lim}} g \ left (x \ right) = A + B [/ latex]
    Различие функций [латекс] \ underset {x \ to a} {\ mathrm {lim}} \ left [f \ left (x \ right) -g \ left (x \ right) \ right] = \ underset {x \ to a } {\ mathrm {lim}} f \ left (x \ right) – \ underset {x \ to a} {\ mathrm {lim}} g \ left (x \ right) = AB [/ latex]
    Произведение функций [латекс] \ underset {x \ to a} {\ mathrm {lim}} \ left [f \ left (x \ right) \ cdot g \ left (x \ right) \ right] = \ underset {x \ to a} {\ mathrm {lim}} f \ left (x \ right) \ cdot \ underset {x \ to a} {\ mathrm {lim}} g \ left (x \ right) = A \ cdot B [/ латекс ]
    Частное функций [латекс] \ underset {x \ to a} {\ mathrm {lim}} \ frac {f \ left (x \ right)} {g \ left (x \ right)} = \ frac {\ underset {x \ в a} {\ mathrm {lim}} f \ left (x \ right)} {\ underset {x \ to a} {\ mathrm {lim}} g \ left (x \ right)} = \ frac {A} {B}, B \ ne 0 [/ латекс]
    Функция, возведенная в степень [латекс] \ underset {x \ to a} {\ mathrm {lim}} {\ left [f \ left (x \ right) \ right]} ^ {n} = {\ left [\ underset {x \ to \ infty} {\ mathrm {lim}} f \ left (x \ right) \ right]} ^ {n} = {A} ^ {n} [/ latex], где [latex] n [/ latex] – это положительное целое число
    n корень -й степени функции, где n – целое положительное число [латекс] \ underset {x \ to a} {\ mathrm {lim}} \ sqrt [n] {f \ left (x \ right)} = \ sqrt [n] {\ underset {x \ to a} { \ mathrm {lim}} \ left [f \ left (x \ right) \ right]} = \ sqrt [n] {A} [/ latex]
    Полиномиальная функция [латекс] \ underset {x \ to a} {\ mathrm {lim}} p \ left (x \ right) = p \ left (a \ right) [/ latex]

    Пример 1: Алгебраическое вычисление предела функции

    Вычислить [латекс] \ underset {x \ to 3} {\ mathrm {lim}} \ left (2x + 5 \ right) [/ latex].

    Решение

    [латекс] \ begin {array} {ll} \ underset {x \ to 3} {\ mathrm {lim}} \ left (2x + 5 \ right) = \ underset {x \ to 3} {\ mathrm {lim }} \ left (2x \ right) + \ underset {x \ to 3} {\ mathrm {lim}} \ left (5 \ right) \ hfill & \ text {Свойство суммы функций} \ hfill \\ \ text { } = \ underset {x \ to 3} {2 \ mathrm {lim}} \ left (x \ right) + \ underset {x \ to 3} {\ mathrm {lim}} \ left (5 \ right) \ hfill & \ text {Постоянное время свойство функции} \ hfill \\ \ text {} = 2 \ left (3 \ right) +5 \ hfill & \ text {Evaluate} \ hfill \\ \ text {} = 11 \ hfill & \ hfill \ end {array} [/ latex]

    Попробуй 1

    Оцените следующий предел: [латекс] \ underset {x \ to -12} {\ mathrm {lim}} \ left (-2x + 2 \ right) [/ latex].

    Решение

    Нахождение предела многочлена

    Не все функции или их пределы включают простое сложение, вычитание или умножение. Некоторые могут включать многочлены. Напомним, что полином – это выражение, состоящее из суммы двух или более членов, каждое из которых состоит из константы и переменной, возведенных в неотрицательную целую степень. Чтобы найти предел полиномиальной функции, мы можем найти пределы отдельных членов функции, а затем сложить их вместе.Кроме того, предел полиномиальной функции, когда [latex] x [/ latex] приближается к [latex] a [/ latex], эквивалентен простой оценке функции для [latex] a [/ latex].

    Как: дана функция, содержащая многочлен, найдите ее предел.

    1. Используйте свойства пределов, чтобы разбить многочлен на отдельные члены.
    2. Найдите пределы отдельных условий.
    3. Сложите пределы вместе.
    4. В качестве альтернативы оцените функцию для [latex] a [/ latex].{3} +5 \ right) [/ латекс].

      Решение

      Нахождение предела мощности или корня

      Когда предел включает степень или корень, нам нужно другое свойство, которое поможет нам его оценить. Квадрат предела функции равен пределу квадрата функции; то же самое и с высшими силами. Точно так же квадратный корень из предела функции равен пределу из квадратного корня из функции; то же самое верно и для высших корней. {5} [/ latex].{2} + 6x + 8} {x – 2} \ right) [/ latex], можем ли мы определить предел функции, когда [latex] x [/ latex] приближается к [latex] a [/ latex]?

      Да. Некоторые функции могут быть перегруппированы алгебраически, чтобы можно было оценить предел упрощенной эквивалентной формы функции.

      Нахождение предела частного

      Нахождение предела функции, выраженной как частное, может быть более сложным. Нам часто нужно переписать функцию алгебраически, прежде чем применять свойства предела.Если знаменатель равен 0, когда мы применяем свойства предела напрямую, мы должны переписать частное в другой форме. Один из подходов – записать частное в факторизованной форме и упростить.

      Практическое руководство. Учитывая предел функции в форме частного, используйте факторинг для ее вычисления.

      1. Полностью разложите на множители числитель и знаменатель.
      2. Упростите, разделив любые множители, общие для числителя и знаменателя.
      3. Оцените полученный предел, не забывая использовать правильный домен.{2} -6x + 8} {x – 2} \ right) = \ underset {x \ to 2} {\ mathrm {lim}} \ left (\ frac {\ left (x – 2 \ right) \ left ( x – 4 \ right)} {x – 2} \ right) \ hfill & \ text {Разложите числитель на множители}. \ hfill \\ \ text {} = \ underset {x \ to 2} {\ mathrm {lim}} \ left (\ frac {\ overline {) \ left (x – 2 \ right)} \ left (x – 4 \ right)} {\ overline {) x – 2}} \ right) \ hfill & \ text {Отмена общие факторы}. \ hfill \\ \ text {} = \ underset {x \ to 2} {\ mathrm {lim}} \ left (x – 4 \ right) \ hfill & \ text {Evaluate}. \ hfill \ \ \ text {} = 2–4 = -2 \ hfill & \ hfill \ end {array} [/ latex]

        Анализ решения

        Когда предел рациональной функции не может быть вычислен напрямую, факторизованные формы числителя и знаменателя могут упроститься до результата, который может быть вычислен.{2} -11x + 28} {7-x} \ right) [/ latex].

        Решение

        Пример 6: Оценка предела частного путем поиска ЖК-дисплея

        Вычислить [латекс] \ underset {x \ to 5} {\ mathrm {lim}} \ left (\ frac {\ frac {1} {x} – \ frac {1} {5}} {x – 5} \ справа) [/ латекс].

        Решение

        Найдите ЖК-дисплей для знаменателей двух членов в числителе и преобразуйте обе дроби, чтобы ЖК-дисплей был их знаменателем.

        Рисунок 3

        Анализ решения

        При определении предела рациональной функции, в которой члены добавляются или вычитаются либо в числителе, либо в знаменателе, первым шагом является поиск общего знаменателя добавленных или вычтенных членов; затем преобразуйте оба члена в этот знаменатель или упростите рациональную функцию, умножив числитель и знаменатель на наименьший общий знаменатель.Затем проверьте, есть ли у числителя и знаменателя общие множители.

        Попробуй 6

        Вычислить [латекс] \ underset {x \ to -5} {\ mathrm {lim}} \ left (\ frac {\ frac {1} {5} + \ frac {1} {x}} {10 + 2x} \ справа) [/ латекс].

        Решение

        Практическое руководство. Учитывая предел функции, содержащей корень, используйте сопряжение для вычисления.

        1. Если данное частное не находится в неопределенной [латексной] \ левой (\ frac {0} {0} \ правой) [/ латексной] форме, оцените напрямую.
        2. В противном случае перепишите сумму (или разность) двух частных как одинарное частное, используя наименьший общий знаменатель (LCD) .
        3. Если числитель включает корень, рационализируйте числитель; умножьте числитель и знаменатель на , сопряженное числителя. Напомним, что [latex] a \ pm \ sqrt {b} [/ latex] являются конъюгатами.
        4. Упростить.
        5. Оцените полученный предел.

        Пример 7: Оценка предела, содержащего корень, с помощью конъюгата

        Вычислить [латекс] \ underset {x \ to 0} {\ mathrm {lim}} \ left (\ frac {\ sqrt {25-x} -5} {x} \ right) [/ latex].

        Решение

        [латекс] \ begin {array} {ll} \ underset {x \ to 0} {\ mathrm {lim}} \ left (\ frac {\ sqrt {25-x} -5} {x} \ right) = \ underset {x \ to 0} {\ mathrm {lim}} \ left (\ frac {\ left (\ sqrt {25-x} -5 \ right)} {x} \ cdot \ frac {\ left (\ sqrt {25-x} +5 \ right)} {\ left (\ sqrt {25-x} +5 \ right)} \ right) \ hfill & \ text {Умножьте числитель и знаменатель на сопряжение}.\ hfill \\ \ text {} = \ underset {x \ to 0} {\ mathrm {lim}} \ left (\ frac {\ left (25-x \ right) -25} {x \ left (\ sqrt { 25-x} +5 \ right)} \ right) \ hfill & \ text {Умножение:} \ left (\ sqrt {25-x} -5 \ right) \ cdot \ left (\ sqrt {25-x} + 5 \ right) = \ left (25-x \ right) -25. \ Hfill \\ \ text {} = \ underset {x \ to 0} {\ mathrm {lim}} \ left (\ frac {-x} {x \ left (\ sqrt {25-x} +5 \ right)} \ right) \ hfill & \ text {Объединить похожие термины}. \ hfill \\ \ text {} = \ underset {x \ to 0} { \ mathrm {lim}} \ left (\ frac {- \ overline {) x}} {\ overline {) x} \ left (\ sqrt {25-x} +5 \ right)} \ right) \ hfill & \ текст {Упростить} \ frac {-x} {x} = – 1.\ hfill \\ \ text {} = \ frac {-1} {\ sqrt {25-0} +5} \ hfill & \ text {Evaluate}. \ hfill \\ \ text {} = \ frac {-1} {5 + 5} = – \ frac {1} {10} \ hfill & \ hfill \ end {array} [/ latex]

        Анализ решения

        При определении предела функции с корнем в качестве одного из двух членов, которые мы не можем вычислить напрямую, подумайте о умножении числителя и знаменателя на сопряжение членов.

        Попробуй 7

        Оцените следующий предел: [латекс] \ underset {h \ to 0} {\ mathrm {lim}} \ left (\ frac {\ sqrt {16-h} -4} {h} \ right) [/ latex] .

        Решение

        Пример 8: Оценка предела частного значения функции путем факторинга

        Вычислить [латекс] \ underset {x \ to 4} {\ mathrm {lim}} \ left (\ frac {4-x} {\ sqrt {x} -2} \ right) [/ latex].

        Решение

        [латекс] \ begin {array} {ll} \ underset {x \ to 4} {\ mathrm {lim}} \ left (\ frac {4-x} {\ sqrt {x} -2} \ right) = \ underset {x \ to 4} {\ mathrm {lim}} \ left (\ frac {\ left (2+ \ sqrt {x} \ right) \ left (2- \ sqrt {x} \ right)} {\ sqrt {x} -2} \ right) \ hfill & \ text {Factor.} \ hfill \\ \ text {} = \ underset {x \ to 4} {\ mathrm {lim}} \ left (\ frac {\ left (2+ \ sqrt {x} \ right) \ overline {) \ left (2- \ sqrt {x} \ right)}} {- \ overline {) \ left (2- \ sqrt {x} \ right) }} \ right) \ hfill & \ text {Factor} -1 \ text {вне знаменателя} \ text {.{2} [/ латекс]

        и может быть разложен на

        [латекс] \ left (a + b \ right) \ left (a-b \ right) [/ латекс].

        Попробуй 8

        Оцените следующий предел: [латекс] \ underset {x \ to 3} {\ mathrm {lim}} \ left (\ frac {x – 3} {\ sqrt {x} – \ sqrt {3}} \ right) [/латекс].

        Решение

        Практическое руководство. Получив частное с абсолютными значениями, оцените его предел.

        1. Попробуйте разложить на множители или найти ЖК-дисплей.
        2. Если предел не может быть найден, выберите несколько значений рядом и по обе стороны от входа, где функция не определена.
        3. Используйте числовое свидетельство, чтобы оценить пределы с обеих сторон.

        Пример 9: Оценка предела частного с абсолютными значениями

        Вычислить [латекс] \ underset {x \ to 7} {\ mathrm {lim}} \ frac {| x – 7 |} {x – 7} [/ latex].

        Решение

        Функция не определена при [latex] x = 7 [/ latex], поэтому мы попробуем значения, близкие к 7 слева и справа.

        Предел слева: [латекс] \ frac {| 6.9 – 7 |} {6.9 – 7} = \ frac {| 6.99 – 7 |} {6.99 – 7} = \ frac {| 6.{+}} {\ mathrm {lim}} \ frac {6-x} {| x – 6 |} [/ latex].

        Решение

        Ключевые понятия

        • Свойства пределов могут использоваться для выполнения операций над пределами функций, а не с самими функциями.
        • Предел полиномиальной функции можно найти, найдя сумму пределов отдельных членов.
        • Предел функции, возведенный в степень, равен той же степени, что и предел функции. Другой метод – прямая подстановка.
        • Предел корня функции равен соответствующему корню предела функции.
        • Один из способов найти предел функции, выраженной как частное, – это записать частное в факторизованной форме и упростить.
        • Другой метод определения предела сложной дроби – найти ЖК-дисплей.
        • Предел, содержащий функцию, содержащую корень, может быть вычислен с помощью конъюгата.
        • Пределы некоторых функций, выраженные в виде частных, можно найти с помощью факторинга.
        • Один из способов оценить предел частного, содержащего абсолютные значения, – использовать числовое свидетельство. Также может быть полезна его настройка по частям.

        Глоссарий

        свойства пределов
        Сборник теорем для нахождения пределов функций путем выполнения математических операций над пределами

        Раздел Упражнения

        1. Приведите пример типа функции [latex] f [/ latex], предел которой, когда [latex] x [/ latex] приближается к [latex] a [/ latex], составляет [latex] f \ left (a \ справа) [/ латекс].

        2. Когда прямая подстановка используется для оценки предела рациональной функции, поскольку [латекс] x [/ латекс] приближается к [латексу] a [/ латексу], и результат [латекс] f \ left (a \ right) = \ frac {0} {0} [/ latex], означает ли это, что предел [latex] f [/ latex] не существует?

        3. Что означает утверждение, что предел [латекса] f \ left (x \ right) [/ latex], поскольку [latex] x [/ latex] приближается к [latex] c [/ latex], не определен?

        Для следующих упражнений оцените пределы алгебраически.

        4.{2}} {x – 2} \ right) [/ латекс]

        Для следующего упражнения используйте данную информацию для оценки пределов: [латекс] \ underset {x \ to c} {\ mathrm {lim}} f \ left (x \ right) = 3 [/ latex], [latex ] \ underset {x \ to c} {\ mathrm {lim}} g \ left (x \ right) = 5 [/ latex]

        31. [латекс] \ underset {x \ to c} {\ mathrm {lim}} \ left [2f \ left (x \ right) + \ sqrt {g \ left (x \ right)} \ right] [/ латекс]

        32. [латекс] \ underset {x \ to c} {\ mathrm {lim}} \ left [3f \ left (x \ right) + \ sqrt {g \ left (x \ right)} \ right] [/ латекс]

        33. [латекс] \ underset {x \ to c} {\ mathrm {lim}} \ frac {f \ left (x \ right)} {g \ left (x \ right)} [/ латекс]

        Для следующих упражнений оцените следующие пределы.{2}} [/ латекс]

        51. [латекс] f \ left (x \ right) = \ sqrt {x} [/ латекс]

        52. Найдите уравнение, которое можно представить на рис. 2.

        Рисунок 2

        53. Найдите уравнение, которое можно представить на рис. 3.

        Рисунок 4

        Следующие упражнения см. На рис. 4.

        Рисунок 5

        54. Каков правый предел функции, когда [latex] x [/ latex] приближается к 0?

        55.{0,0425t} [/ latex], где [latex] {A} _ {0} [/ latex] – начальная сумма инвестиций. Найдите среднюю скорость изменения баланса счета с [latex] t = 1 [/ latex] года до [latex] t = 2 [/ latex] лет, если первоначальная сумма инвестиций составляет 1000,00 долларов США.

        Эпсилон-дельта Определение предела

        В общем, чтобы доказать предел с помощью техники ε \ varepsilonε-δ \ deltaδ, мы должны найти выражение для δ \ deltaδ, а затем показать, что требуемые неравенства выполняются. Выражение для δ \ deltaδ чаще всего выражается через ε, \ varepsilon, ε, хотя иногда оно также является константой или более сложным выражением.Ниже приведены несколько примеров, демонстрирующих это свойство.

        Покажите, что lim⁡x → πx = π. \ Displaystyle \ lim_ {x \ rightarrow \ pi} x = \ pi.x → πlim x = π.


        Пусть f (x) = x.f (x) = x.f (x) = x. Сначала нам нужно определить, какое значение будет иметь наше δ \ deltaδ. Когда ∣x − π∣ <δ \ vert x - \ pi \ vert <\ delta∣x − π∣ <δ, мы хотим, чтобы ∣f (x) −π∣ <ε. \ Vert f (x) - \ pi \ vert <\ varepsilon.∣f (x) −π∣ <ε. Мы знаем, что ∣f (x) −π∣ = ∣x − π∣ <δ, \ vert f (x) - \ pi \ vert = \ vert x - \ pi \ vert <\ delta, ∣f (x) - π∣ = ∣x − π∣ <δ, поэтому, взяв δ = ε \ delta = \ varepsilonδ = ε, мы получим желаемое свойство.

        Есть другие значения δ \ deltaδ, которые мы могли бы выбрать, например, δ = ε7. \ Delta = \ frac {\ varepsilon} {7} .δ = 7ε. Почему это значение δ \ deltaδ также было бы приемлемым? Если ∣x − π∣ <δ = ε7, \ vert x - \ pi \ vert <\ delta = \ frac {\ varepsilon} {7}, ∣x − π∣ <δ = 7ε, то ∣f (x) −π∣ <ε7 <ε, \ vert f (x) - \ pi \ vert <\ frac {\ varepsilon} {7} <\ varepsilon, ∣f (x) −π∣ <7ε <ε, если требуется. □ _ \ квадрат □

        Чтобы показать, что предел существует, нам не обязательно доказывать, что результат верен для всех ε, \ varepsilon, ε, но достаточно показать, что результат верен для всех ε e. \ Varepsilon> e.ε> e.

        Покажи, что

        lim⁡x → 1 (5x − 3) = 2. \ Lim _ {x \ to 1} {(5x-3)} = 2.x → 1lim (5x − 3) = 2.


        В этом примере мы имеем x0 = 1x_ {0} = 1×0 = 1, f (x) = 5x − 3f (x) = 5x -3f (x) = 5x − 3 и L = 2L = 2L = 2 из определения предела, данного выше. Для любого ε> 0 \ varepsilon> 0ε> 0, выбранного Алисой, Боб хотел бы найти такое δ> 0 \ delta> 0δ> 0, что если xxx находится на расстоянии δ \ deltaδ от x0 = 1x_ {0} = 1×0 = 1, п.е.

        x − 1∣ <δ, \ влево | x-1 \ right | <\ delta, ∣x − 1∣ <δ,

        , то f (x) f (x) f (x) находится на расстоянии ε \ varepsilonε от L = 2L = 2L = 2, т.е.

        ∣f (x) −2∣ <ε. \ Left | f (x) - 2 \ right | <\ varepsilon.∣f (x) −2∣ <ε.

        Чтобы найти δ \ deltaδ, Боб работает в обратном направлении от неравенства ε \ varepsilonε:

        ∣ (5x − 3) −2∣ = ∣5x − 5∣ <ε = 5∣x − 1∣ <ε = ∣x − 1∣ <ε5. \ Begin {выровнено} \ left | (5x-3) - 2 \ вправо | & = \ left | 5x-5 \ right | <\ varepsilon \\ & = 5 \ влево | x-1 \ вправо | <\ varepsilon \\ & = \ left | x-1 \ right | <\ frac {\ varepsilon} {5}.\ end {align} ∣ (5x − 3) −2∣ = ∣5x − 5∣ <ε = 5∣x − 1∣ <ε = ∣x − 1∣ <5ε.

        Итак, Боб дает Алисе значение δ = ε5 \ delta = \ frac {\ varepsilon} {5} δ = 5ε. Затем Алиса может проверить, что если ∣x − 1∣ <δ = ε5 \ left | x-1 \ right | <\ delta = \ frac {\ varepsilon} {5} ∣x − 1∣ <δ = 5ε, тогда

        ∣ (5x − 3) −2∣ = ∣5x − 5∣ = 5∣x − 1∣ <5 (ε5) = ε. □ \ left | (5x-3) -2 \ right | = \ влево | 5x-5 \ вправо | = 5 \ влево | x-1 \ вправо | <5 \ left (\ frac {\ varepsilon} {5} \ right) = \ varepsilon. \ _ \ square∣ (5x − 3) −2∣ = ∣5x − 5∣ = 5∣x − 1∣ <5 (5ε) = ε. □

        Докажите, что lim⁡x → 7 (x2 + 1) = 50.2 – 49 \ верт \\ & = \ верт х-7 \ верт \ верт х + 7 \ верт \\ & <\ delta \ vert x + 7 \ vert. \ end {array} ∣x2 + 1−50∣ = ∣x2−49∣ = ∣x − 7∣∣x + 7∣ <δ∣x + 7∣.

        Предполагая ∣x − 7∣ <1, \ vert x - 7 \ vert <1, ∣x − 7∣ <1, имеем wex∣ <8, \ vert x \ vert <8, ∣x∣ <8, откуда следует ∣x + 7∣ <∣x∣ + ∣7∣ = 15 \ vert x + 7 \ vert <\ vert x \ vert + \ vert 7 \ vert = 15∣x + 7∣ <∣x∣ + ∣7 ∣ = 15 по неравенству треугольника.

        Итак, когда мы полагаем δ = min⁡ (1, ε15), \ delta = \ min \ left (1, \ frac {\ varepsilon} {15} \ right), δ = min (1,15ε), мы будет

        ∣x2 + 1−50∣ <δ∣x + 7∣ <15δ <ε.2 + 1 - 50 \ vert & <\ delta \ vert x + 7 \ vert \\ & <15 \ дельта \\ & <\ varepsilon. \ _\квадратный \ end {array} ∣x2 + 1−50∣ <δ∣x + 7∣ <15δ <ε. □

        Используя определение ε \ varepsilonε-δ \ deltaδ, докажите следующий предел:

        lim⁡x → 0sin⁡xx = 1. \ lim_ {x \ rightarrow 0} \ frac {\ sin x} {x} = 1.x → 0lim xsinx = 1.


        На интервале (−π2, π2) \ left (- \ frac {\ pi} {2}, \ frac {\ pi} {2} \ right) (−2π, 2π) имеем 0 < cos⁡x

        ∣sin⁡xx − 1∣ <1 − cos⁡x∣sin⁡xx − 1∣ <2sin⁡2x2

        , что завершает доказательство. Следовательно,

        lim⁡x → 0sin⁡xx = 1. □ \ lim_ {x \ rightarrow 0} \ frac {\ sin x} {x} = 1. \ _ \ squarex → 0lim xsinx = 1. □

        Есть случаи, когда предел f (x) f (x) f (x) по мере приближения xxx к x0x_0x0 неограниченно увеличивается или уменьшается. В таких случаях часто говорят, что предел существует и значение равно бесконечности (или отрицательной бесконечности). Однако некоторые ресурсы говорят, что в данном случае ограничения не существует просто потому, что это ограничение упрощает формулирование и запоминание других теорем в исчислении.2} = \ infty. \ _ \ squarex → 0lim x21 = ∞. □

        .

        Оставить комментарий